You are on page 1of 70

Pulp and periapical diseases

1. Patient came to do RCT, after cleaning and shaping there is drainage, how to manage ?
A. keep it open
B. put temporary restoration
C. put composite
D. do perforation

2. 30-year-old patient has sinus drain (chronic) and Not responding (non vital) to cold test
with slight tender to percussion (apical periodontitis) What is the diagnosis?
A. irreversible pulpitis normal apical issue
B. irreversible pulpitis chronic apical abscess
C. necrotic pulpitis asymptomatic periodontitis
D. Necrosis pulp with chronic apical tissue
 Sinus drain = chronic.
 No response to cold (pulp test) = necrotic.
Pain on percussion = periapical issue or periodontitis
3. pt with swelling in the face started before 2 days ago, has a tooth with no response to cold
and tender to percussion in radiograph no radiolucency diagnosis ?
A. acute abscess
B. chronic abscess
C. necrotic with symptomatic periodontitis
D. necrotic with asymptomatic periodontitis

4. what is the most cause necrosis?


A. Avulsion
B. Extrusion
C. subluxation
D. concussion
 between the options…. (Concussion least, Most= intrusion if in the options)

5. Intrinsic stains after RCT due to?


A. Trauma to dentinal tubules
B. blood from dentinal tubules
C. restoration remnant
6. Which of the following is the endodontic emergency treatment for teeth with symptomatic
irreversible pulpitis ?
A. Pulpectomy
B. trephination
C. direct pulp capping
D. incision and drainage
7. Method to assess blood flow in pulp?
A. Laser oximetry
B. doppler flowmetry
1
 Pulse oximetry measure= oxygen concentration in blood
 Doppler flowmetry = assess blood flow
8. 30-year-old patient has sinus drain and Not responding to cold test with slight tender to
percussion What is the diagnosis?
A. irreversible pulpitis; normal apical tissue
B. irreversible pulpitis; chronic apical abscess
C. necrotic pulp; asymptomatic periodontitis
D. Necrosis pulp with chronic apical periodontitis
9. Most characteristic of irreversible pulpitis?
A. pain on cold, (Hypersensitivity)
B. Lingering pain on cold
C. pain in percussion (periapical area)

10. patient complain from his lower molar Xray show: lower molar with crown and
radiolucency in the apex and the lateral distal surface of the root. Pain on percussion,
pocket depth distally: more than 10mm, cold test: negative ?
A. subgingival calculus
B. lateral periodontal abscess
C. asymptomatic periapical periodontitis
11. most common organism found in chronic abscess?
A. Aerobic
B. Anaerobic
C. Mixed
 Acute endo ->anaerobic bacteria
 Chronic endo -> mixed bacteria
12. Definition of endodontic flare up?
A. Acute exacerbation of periradicular pathosis after start of RCT

A phoenix abscess is an acute exacerbation of a chronic periapical lesion. It is a dental


abscess that can occur immediately following root canal treatment.
Another cause is due to untreated necrotic pulp (chronic apical periodontitis).
It is also the result of inadequate debridement during the endodontic procedure.
13. A 20-year-old patient is complaining of Pain in the lower right area. Upon examination, #47
responded normally to vitality testing, #46 is signs of a sinus tract or swelling. Radiograph
provided (see image) Which of the following is the appropriate treatment plan for tooth #46?
A. Extraction
B. Hemisection
C. Apicoectomy
D. Non-surgical retreatment

2
14. A 29-year-old man came to the clinic complaining of severe pain related to his upper left
molar. He pointed to tooth #16 that was endodontically treated a couple of months ago. The
tooth has a crown with a good margin and no recurrent decay. The tooth has severe pain with
percussion and palpation. Probing depths were with normal limits. PA radiograph was taken
(see image) Which of the following is the next step to know the exact cause of the patient?
(phoenix abscess)
A. Sinus tract tracing
B. Intentional replantation
C. Cone beam computed tomography
15. A 57-year-old patient presented for root canal treatment of tooth 16 upon examination,
tooth responded normally to percussion and palpation, negative to cold and EPT Which
of the following is the most diagnosis of tooth #16?
A. Necrotic pulp with normal periapical tissue
B. Previously initiated with normal periapical tissue
C. Necrotic pulp with symptomatic apical periodontitis
D. Previously treated with symptomatic apical periodontitis

16. A 49-year-old woman complaining of sensitivity to both hot and cold liquids following
the placement of a full crown on tooth #17 which recently became spontaneous.
Responses to both percussion and palpation were normal and on application of Endo-Ice
the patient experienced pain that lingered after removal of the stimulus. Radiographically,
there was no evidence of osseous change Which of the following is the pulpal and
periapical diagnosis of this tooth?
A. Irreversible pulpitis; normal apical tissue
B. Irreversible pulpitis; chronic apical abscess
C. Necrotic pulp; asymptomatic periodontitis
D. Reversible pulpitis; symptomatic apical periodontitis

17. A 59-year-old patient complaining of a pimple on his gum in the upper right area, tooth #17
does not response to cold and there is no tenderness to biting or percussion. A sinus tract is
present opposite to the upper right molar. Radiograph provided Which of the following is the
recommended treatment plan for tooth #17?
A. Extraction
B. Biopsy of the lesion
C. Root canal treatment
D. Incision and drainage

3
18. A 36-year-old man came to the clinic with a chief complain of mild persistent pain on
biting related to tooth #15. It has previous endodontic treatment Bitewing radiograph:
shows a radiolucent line below filling margin indicating recurrent decay. Periapical
radiograph: shows a short radicular filling, obturation and periapical radiolucency related
to the apex of tooth #15. Probing depths are within normal limits. Which of the following
is the cause for failure?
A. Root fracture
B. Persistent lesion
C. Persistent bacteria in the apical lesion
D. Poor root canal treatment and coronal leakage
19. A 27-year-old asthmatic woman presents with pain on tooth #24 which is aggravated by
touch. The tooth was endodontically treated 2 years ago, but the tooth is still
symptomatic. Tooth #24 is negative to cold and EPT, and sensitive to percussion (see
image) Which of the following would be the best management for tooth #24?
A. Extraction and implant
B. Surgical endodontic treatment
C. Non-surgical endodontic treatment
D. Prescription of clindamycin antibiotic
20. Which of the following is the endodontic emergency treatment for teeth with
symptomatic irreversible pulpitis?
A. Pulpotomy
B. Trephination
C. Direct pulp capping
D. Incision and drainage
21. A 65-year-old patient complaining of a spontaneous dull pain in the upper right side tooth
#17 tender to biting and percussion. No clinical signs of sinus tract or swelling is present.
Radiograph provided (see image). Which of the following is the endodontic diagnosis of
tooth #17?
A. Pulp necrosis with asymptomatic apical periodontitis
B. Asymptomatic irreversible pulpitis with acute apical abscess
C. Symptomatic irreversible pulpitis with chronic apical abscess
D. Previously treated with symptomatic periodontitis
Ideal answer is: chronic pulpities with symptomayic apical periodontities.

4
22. A 40-year-old patient presented with localized swelling in the area of tooth #16.
Diagnosis was necrotic pulp with acute apical abscess. Patient does not have time to stay
for full treatment (see image) Which of the following is the appropriate emergency
treatment for tooth #16?
A. Extraction
B. Antibiotics
C. Incision and drainage
D. Cleaning and shaping
23. A 27-year-old healthy woman presents to the dental clinic for checkup. Upon
examination, tooth #16 did not respond to thermal and electrical stimulation. A sinus tract
was noted in the attached gingival between #16 and #17. All upper right premolar and
molar teeth responded normally to percussion and palpation tests. Periodontal probing
ranged between 2 – 3 mm (image) Which of the following is the recommended
management for tooth #16?
A. Tooth extraction
B. Root canal treatment
C. Excision of sinus tract
D. Prophylactic antibiotic

24. 27-year-old woman presents for dental check-up. Patient mentioned a history of traumatic
blow to the lower right premolar area received about 4 years ago. Tooth #44 did not
respond to thermal and electric pulp stimulation. Palpation, percussion and periodontal
tests were normal. No swelling or sinus tract were noted. Patient is keen to save the tooth
if possible (see image) Which of the following is the most appropriate management for
tooth #44?
A. Pulpotomy
B. Apical curettage
C. Extraction and implant
D. RCT
25. A 22-year-old woman was seen in clinic for a dental abscess in the mandible due to no35.
She was treated with oral antibiotics. She returned to the clinic with persistent abscess
and pain. Which of the following could have prevented the persistence of the infection?
A. Prescribe mouthwash
B. Incisional and drainage
C. Endodontic treatment
D. Extract the offending tooth
26. A patient came to the clinic suffering from intense pain from tooth #46. The tooth
showed severe tenderness to percussion and biting. What is most appropriate periapical
diagnosis?
A. Condensing osteitis
B. Chronic apical abscess
5
C. Symptomatic apical periodontitis
D. Asymptomatic apical periodontitis
27. A 47-year-old patient referred to Endodontic Clinic evaluate tooth #45. Upon
examination, tooth #45 had root canal treatment 3 years ago and is sensitive to percussion
(see image) Which of the following is diagnosis of tooth #45?
A. Previously treated with normal apical abscess
B. Previously treated with acute apical abscess
C. Previously treated with symptomatic apical periodontitis
D. Previously treated with asymptomatic apical periodontitis
28. A 50-year-old woman presents to the dental clinic for evaluation of upper left premolar-
molar teeth. Patient stated that she had a severe spontaneous pain 6 weeks ago while on
holiday. The pain has eventually subsided without dental intervention. Teeth #24,26 and
27 responded normally to all endodontic tests. Tooth #25 did not respond to cold. A sinus
tract was noted in the alveolar mucosa buccal to #25 e image Which of the following is
the endodontic diagnosis for tooth #25?
A. Pulp necrosis with chronic apical abscess
B. Previously treated with acute apical abscess
C. Symptomatic irreversible pulpitis with symptomatic apical
Sinus tract: chronic case, no pulp response: necrosis.

29. A 36-year-old patient-complaining of sharp spontaneous pain waking him up at night.


Tooth 47 responds Lingering pain and tender to neither percussion nor biting No sinus
tract or swelling present clinically. Radiography provided (see image) Which of the
following is the endodontic diagnosis of tooth # 47?
A. Reversible pulpitis with acute apical abscess
B. Pulp necrosis with asymptomatic apical periodontitis
C. Symptomatic irreversible pulpitis with normal periapical area.
D. Asymptomatic irreversible pulpitis with chronic apical abscess

30. A 60 years old patient presented with intermittent pain in tooth #26 that is associated with
sinus tract upon examination, tooth #26 is sensitive to percussion and palpation,7mm
probing depth in mesiopalatal area of the tooth and grad II mobility. Which of the
following is the most probable diagnosis of tooth #26?
A. Previously treated with acute apical abscess
B. Previously treated associated with vertical root fracture
C. Previously treated associated symptomatic apical periodontitis
D. Previously treated with asymptomatic apical periodontitis

6
31. 42-year-old-healthy woman presents with moderate pain to cold drinks and sweets in the
upper left side of the jaw. On examination, Application of Endo Ice to tooth #25 elicited a
sharp, momentary pain. Tooth #25 responds normally to percussion, while #26 is
percussion positive (see image) Which of the following is the source of patient
complaint?
A. Pulpal tissues of tooth #25
B. Periapical tissues of tooth #25
C. Pulpal tissues of tooth #26
D. Periapical tissues of tooth #26
Note: the Q didn’t mention anything about pulp test to 26, so we consider source of
diffused pain related to pulp condition of 25.

32. A 63-year-old patient complaining throbbing pain in the upper left area, tooth #26 does
not respond to cold and is tender to both percussion and biting No clinical signs of
swelling or sinus tract are present. Radiograph provided (see image) Which of the
following is the endodontic diagnosis of tooth #26?
A. Pulp necrosis with symptomatic apical periodontitis
B. Symptomatic irreversible pulpitis with normal apical tissue
C. Asymptomatic irreversible pulpitis with chronic apical abscess
D. Previously initiated therapy with asymptomatic apical periodontitis

33. A 43-year-old patient presented to the dental clinic with severe lingering pain related to
the mandibular right posterior teeth. The pain was tolerable until last night, which was
severe interfered with sleeping. Clinically, all teeth were negative to percussion and
palpation, periodontally sound. The cold test revealed an exaggerate lingering response in
the first molar. while a sharp momentary response disappeared after stimulus removal on
the second premolar and second molar (see image) Which of the following treatments
would resolve patient symptoms?
A. RCT of the first molar
B. RCT of the second premolar
C. Direct pulp capping of the first molar
D. Indirect pulp capping of the second molar

7
34. A 64-year-old man presents to the dental clinic complaining that he cannot chew on the
lower right side of the jaw. Pain started 4 weeks ago and has intensifies over the last 3
days. On examination, application of cold to tooth #46 elicited an extraction in that lasted
for about 1 minute. Tooth tender Adjacent premolar and molar teeth responded normally
to pulpal, periapical and periodontal tests (see image) Which of the following is the
endodontic diagnosis for tooth #46?
A. Pulp necrosis with acute apical abscess
B. Reversible pulpitis with symptomatic apical periodontitis
C. Asymptomatic irreversible pulpitis with symptomatic apical periodontitis
D. Symptomatic irreversible pulpitis with symptomatic apical periodontitis

35. How can a periodontal abscess be definitely differentiated from an endodontic abscess?
A. Location
B. Pain source
C. Pulp vitality
D. Radiographic appearance

36. A 61-year-old man presents with severe sensitivity to cold drinks in the lower left molar
region. On examination, application of Endo Ice to tooth #37 produced a strong pain that
lasted for 45 seconds, while percussing the tooth produced a sensitive response. Which of
the following is the endodontic diagnosis for #37
A. Reversible pulpitis with normal apical tissues
B. Necrotic pulp with symptomatic apical periodontitis
C. Necrotic pulp with asymptomatic apical periodontitis
D. Symptomatic irreversible pulpitis with symptomatic apical periodontitis
37. A 52-year-old patient complaining of pain in the upper right area. Tooth #17 did not
respond to cold, and is tender to biting and percussion, no sinus tract or swelling is
present, radiograph provided. Which of the following is the endodontic diagnosis of tooth
#27?
A. Reversible pulpitis with normal apical tissues
B. Pulp necrosis with symptomatic apical periodontitis
C. Symptomatic irreversible pulpitis with chronic apical abscess
D. Asymptomatic irreversible pulpitis with acute apical abscess

38. A patient presents to the dentist complaining of severe unlocalized pain on lower right
side. On clinical examination, he had multiple questionable teeth. An attempt was made
to localize the offending tooth using thermal testing. Tooth #45 had severe pain that
lasted 1-2 seconds after removal of the stimulus. Which of the following is the most
likely diagnosis?
8
A. Normal
B. Necrotic pulp
C. Reversible pulpitis
D. Irreversible pulpitis

39. A 30-year-old woman complains of throbbing pain in the maxillary right quadrant. The
pain is spontaneous and lasts for hours. All tests are normal expect radiographs (see
report). Radiographic examination: showed large carious lesion in the maxillary first
premolar. Pulpal exposure of 2 mm is encountered after excavating all caries. Which of
the following is the most likely diagnosis?
A. Necrosis
B. Reversible pulpitis
C. Irreversible pulpitis
D. Hyperplastic pulpitis

40. What is the diameter size of 3 Gates Glidden in millimeter?


A. 0.5
B. 0.7
C. 0.9
D. 1.1
Note: The diameter of the cutting bud on the GGD #1 is 0.5 mm, #2 is 0.7 mm, #3 is 0.9 mm, #4
is 1.10 mm, #5 is 1.3 mm and #6 is 1.5 mm.
41. Which of the following irrigants has the highest antimicrobial activity against endodontic
infection?
A. EDTA
B. Cetrimide
C. Chlorhexidine
D. Sodium hypochlorite
42. Gutta-percha is currently the standard and universally accepted core obturating material
in most of root canal treatments. Which of the following is disadvantage of this material?
A. Not easily manipulated
B. Difficult to be removed from a canal
C. Shrinkage if softened by heat or solvents
D. Poor adaptation to canal wall with compaction
43. Which of the following is the cross-section of the Flax-R file?
A. Round
B. Square
C. Triangular
D. Quadrangular

9
44. Which one removes the smear layer ?
A. ETDA
B. Sodium hypochlorite
C. Chlorhexidine

45. Best location & size of root perforation that is favorable?


A. Small perforation below height of bone crest
B. Large perforation below height of bone crest
C. Small perforation at height of bone crest
D. Large perforation at height of bone crest

 Perforation near cervical → Poor prognosis. Perforations in the coronal third of the
root adjacent to the crestal alveolar bone have the worst prognosis compared to
perforations in the other areas of the root. If left untreated, apical migration of the
periodontal attachment (i.e., pocket formation) happens relatively quickly. Once the
pocket is formed, persistent inflammation of the area is expected due to continuous
bacterial ingress through the pocket .
 Perforation near apical → Good prognosis.
 Perforation subcrestal→ Bad prognosis. Can’t be achieved for repairing.
 Perforation supracrestal→ Good prognosis better accessibility for repair.

46. Best way to apply sealer into the canal?


A. Files
B. Peeso reamer
C. Lentulo spiral (Lentulo spiral instrument used to spread sealer in the canal )

10
47. What does a localized periodontal defect around an endodontically treated tooth suggest?
A. Gingivitis
B. Internal resorption
C. External resorption
D. Vertical root fracture
 J-shaped radiolucency around endodontically treated tooth =

 VRF

 Techniques for detecting vertical crown/root fractures:

In vital teeth the most common reason for a fracture is trauma. In non-vital teeth trauma may
also be a contributory factor but endodontic treatment followed by post reinforcement is a
common cause.
 1. Fibre optic examination.
 2. Wedging and staining .
 3. Radiographic examination.
 4. Thorough dental history.
 5. Bite tests.
 6. Periodontal probing.
 7. Restoration removal.
48. How is a vertical root fracture best confirmed?
A. Bite test
B. Percussion
C. Exploratory surgery
 Best radiograph for diagnosis of root fracture cbct.
 Best method to diagnose root fracture: exploratory surgery

49. Patient have bone lose in lower 6 it's about 7mm in mesial side, so the diagnosis is ?
A. Hemi section
B. Vertical fracture
C. RCT
D. Extraction

50. 60 years old patient presented with intermittent pain in tooth #26 that is associated with
sinus tract upon examination, tooth #26 is sensitive to percussion and palpation,7mm
probing depth in the mesiopalatal area of the tooth and grade II mobility. Which of the
following is the most probable diagnosis of tooth #26?

11
A. Previously treated with acute apical abscess
B. Previously treated associated with vertical root fracture
C. Previously treated associated without vertical root fracture
D. Previously treated with asymptomatic apical periodontitis

51. A 22 - year-old man presented to the clinic complaining of shooting pain on biting his
maxillary left tooth, which subsided as soon as he released the bite. On clinical
examination, tooth # 25 had a large disto-occlusal composite filling, and was tender to
both vertical and horizontal percussion. The patient mentioned that he was dieting and
frequently ate nuts. The gingival tissue around the tooth was healthy. There were other
teeth in his mouth that required simple filling (see report) Radiographic examination:
showed that the tooth was root canal treated and the restoration margins were intact.
Which of the following is the most likely diagnosis of this tooth ?
A. Cracked tooth
B. Root resorption
C. Periapical abscess
D. Periapical periodontitis
52. A 22-year-old patient has upper #12 fractures with pinhole pulp exposure 12? before 1
hour, what is the Tx?
A. RCT (more than 1 hour)
B. Direct pulp capping
C. Pulpotomy
 Note: According to the time we will decide, if less than 1 h= DPC (Direct Pulp
Capping) or pulpotomy + Depending on size of pinhole < 0.5
53. Crown fracture involving enamel, dentin and exposing the pulp, classify?
A. Crown infarction ( Def: shallow crack that is entirely contained within the enamel and
does not continue into the dentin )
B. complicated crown fracture
C. uncomplicated crown fracture
 Uncomplicated involving enamel +dentin
 Complicated involving enamel +dentin +pulp

54. what is your diagnosis for this picture

A. internal resorption
B. external resorption

12
Pink tooth: This pink discoloration is caused by the dentine being replaced internally by
soft ('fibro-vascular') tissue with the overlying shell of enamel remaining intact.

55. location of second canal in lower canine ?


A. Mesial
B. Distal
C. Labial
D. Lingual

56. In the mid of root suddenly the canal disappears means?


A. Second canal (in x-ray)
B. Internal resorption
C. calcified canal

A “fast break” is a term used to describe a situation where a root canal disappears on a
radiograph as you move apically. This happens when the main canal splits into
multiple smaller canals that are not discernible on a radiograph.

57. Dentist doing RCT, suddenly bleeding occurred through the canal. The dentist noticed
that the rubber stopper on the file has moved from it's original location? What is
happened?
A. Internal resorption
B. External resorption
C. Perforation
D. Ledge
58. Pt came on a routine checkup. Dentist noticed an overextended cone 5 mm on #14.
Patient had his RCT 10 years ago. Asymptomatic no signs of peri apical pathosis on x-
ray. Management?
A. Follow up.
B. Surgical removal of the extended cone
59. A 30-year-old patient complains of severe pain in #36, x-rays were performed and it was
found that the canal curved. What is the appropriate system?
A. rotary
B. hand instrument only
C. both
D. avoid Gates glidden

13
60. Which tooth constant number of canal ?
A. Maxillary central incisor
B. Mandibular central incisor
C. Maxillary canine
D. Mandibular canine
61. Which of the following canals in(6) is most difficult to locate?
A. palatal.
B. Distobuccal.
C. Mesiobuccal.
62. what is the most cause necrosis?
E. Avulsion
F. Extrusion
G. subluxation
H. concussion
 between the options…. (Concussion least, Most= intrusion if in the options)
63. Intrinsic stains after RCT due to?
D. Trauma to dentinal tubules
E. blood from dentinal tubules
F. restoration remnant
64. Carrier based technique ?
A. Obtera 3
B. Thermafil

Note:
 Thermomechanical compaction = McSpadden Compactor
compactor is used which resembles a reverse H-file. A cone coated with sealer is
placed in the root canal, engaged with a rotary instrument (running between 5,000 and
10,000 rpm) that frictionally warms, plasticizes and compacts it into the root canal.

 thermoplastic injection technique = Obtura II (internal resorption)


Obtura II is a popular warm gutta-percha technic that uses a "gun" to warm and inject
the gutta-percha filling into the root canal.

14
 warm vertical compaction + continuous wave compaction technique =System B

 Carrier-Based Gutta-Percha = thermafil + pro taper + SuccessFil case

65. System-B obturation technique is?


A. Cold GP. Condensation
B. Warm GP. Condensation
C. Continuous condensation
System B
Continuous wave of condensation is an obturation technique that applies a heated “plugger”
into a single custom fit greater tapered master cone of gutta-percha.
The heat plugger is placed through the master cone to within 3-5mm of the predetermined
working length.
66. Best material to repair Perforation in furcation ?
A. GIC

15
B. Zinc oxide eugenol
C. Calcium hydroxide
D. Bio-ceramics

67. Where is the narrowest diameter in root?


A. Anatomical apex
B. Radiographic apex
C. Physiological apex
D. apical constriction

68. The pt have PFM crown and he needs RCT but he doesn't want to remove the crown, the
access cavity will be done by ?
A. Carbide bur
B. Dimond bur
C. Diamond for poreclin and transmetal bur for metal
69. Pulp exposure in trauma for 3 days?
A. Pulpectomy
B. Pulp capping
C. Cervical pulpotomy.
D. Pulpotomy
70. Negative rake angle in ?
A. K-files
B. H- files
 K file -ve rake angle. H file +ve rake angle
71. The length of working part in file 20 is?
A. 12
B. 16

16
C. 21
72. Which of the following is the endodontic emergency treatment for teeth with symptomatic
irreversible pulpitis ?
A. Pulpectomy
B. trephination
C. direct pulp capping
D. incision and drainage

73. Which of the following is an error occurs during obturation of the root canal ?
A. voids (improper compaction).
B. ledge formation
C. broken instrument
D. filling lateral canal
74. What does a localized periodontal defect around an endodontically treated tooth suggest?
A. gingivitis
B. internal resorption
C. external resorption
D. vertical root fracture
75. What is the use of root canal Sealer in endodontics?
A. Fill the voids
B. Increase strength of RC filling
C. Disinfect the canal
D. Irrigation of canal
76. percentage of gutta percha in composition?
A. 100% gutta percha
B. 50%gutta percha 50% wax
C. 70% gutta percha 30% zinc oxide
D. 20%gutta percha 80%zinc oxide

77. Root Resorption inhibition property after replantation of avulsed teeth?


A. Edta
B. Ledermix
C. Sodium perborate

17
LEDERMIX dental cement powder contains the active ingredients demeclocycline
hydrochloride and triamcinolone acetonide. is a corticosteroid used to reduce swelling or
inflammation. tooth, and as a dressing during the treatment of the root canal. incomplete
development of hard tooth surface.

78. Material used for Apexification?


A. Caoh
B. mta

79. Conditioning of resected root end by ?


A. Citric acid.
B. EDTA!!!!
C. Phosphoric acid.
D. H202

80. best storage medium for avulsed teeth ?


A. Hbss (Hanks' Balanced Salt Solution)
 from best to worst media: 1-cell culture media HBSS 2-low fat cold milk 3-
physiological saline. 4-Saliva 5-water

81. Coronal Lateral perforation observed with no. 15 K file small in size and supracrustal
location, managed with MTA, what's the prognosis?
A. Good
B. Bad

Factors Affecting Prognosis of Perforation Repair:

 Visibility and accessibility – Better the access and visibility to the site of perforation,
better is the clinician’s ability to repair it
 Time – Faster the perforation repair better is the prognosis

18
 Esthetics – It is to be considered when the perforation repair is in case of anterior or
premolar teeth
 Associated periodontal condition
 Location –
 Perforation in the furcation area has a poor prognosis.
 Perforation occurring in mid-root and apical part of root does not have communication
with oral cavity and thus has good prognosis.
 Size - A smaller perforation has less tissue destruction and inflammation, thus having
better prognosis than larger sized perforation

Most common anatomic site in the tooth where perforation occurs:

 Maxillary incissors – facial aspect


 Mandibular incissors – facial, mesial or distal aspect
 Maxillary premolars – mesial or distal aspects
 Mandibular premolars – facial aspect
 Mandibular molars – lingual aspect
 Maxillary and mandibular molars with receeded pulp chamber – furcation area

It can be classified

 Cervical canal perforation occurs mainly during location of canal orifice and flaring of
the coronal third of the root canal
 Mid-root perforation occurs due to over instrumentation and over-preparation of thin
wall of root or concave side of the curved canals
 Apical root perforation occurs when instrument goes beyond the confines of the root
canal and by overuse of chelating agents along with straight and stiffer large-sized
instruments to negotiate ledge, canal blockage, or zipping, etc.

82. Method to assess blood flow in pulp?


C. Laser oximetry
D. doppler flowmetry
 Pulse oximetry measure= oxygen concentration in blood
 Doppler flowmetry = assess blood flow
83. Instrument fracture at middle part, management ?
A. Bypass
B. Retrieve it
 Instrument fracture and management:
 Apical = RCT and follow up
 Middle =bypass
 Coronal =retrieve it
84. Yellowish discoloration of tooth after RCT due to?
A. Hemorrhage from pulp
B. Incomplete debridement of pulp tissuev

19
C. Leaving Gutta percha in pulp chamber
D. Hemorrhage from canal + leaving Gutta percha

85. A 22 - year-old man presented to the clinic complaining of shooting pain on biting his
maxillary left tooth, which subsided as soon as he released the bite. On clinical
examination, tooth # 25 had a large disto-occlusal composite filling, and was tender to
both vertical and horizontal percussion. The patient mentioned that he was dieting and
frequently ate nuts. The gingival tissue around the tooth was healthy. There were other
teeth in his mouth that required simple filling (see report) Radiographic examination:
showed that the tooth was root canal treated and the restoration margins were intact.
Which of the following is the most likely diagnosis of this tooth ?
E. Cracked tooth
F. Root resorption
G. Periapical abscess
H. Periapical periodontitis

Note:
- Vertical percussion: periapical area.
- Horizontal percussion: periapical area.
Cracked tooth syndrome: defined as a fracture plane of unknown depth, which originate
from the crown, passes through the tooth structure and extends subgingivally, and may
progress to connect with the pulp space and/or periodontal ligament.

When the fractured portions of the tooth move independently of each other, it causes
sudden movement of fluid present in the dentinal tubules.
This causes activation of myelinated A-type fibers within the dental pulp and results in
acute pain.

20
Hypersensitivity to cold may occur due to the seepage of toxic irritants through the crack.
This leakage of toxic irritants causes the release of neuropeptides, and a concomitant
lowering in the pain threshold of unmyelinated C-type fibers within the dental pulp.

Diagnosis:
Dye test: Special stains such as methylene blue or gentian violet are frequently used to highlight
the cracks

 Periodontal probing: Cracked tooth and a split tooth may be differentiated by


periodontal probing. The localized periodontal defect is the result of a fracture line
extending below the gingiva.

 Bite tests: Pain on biting that ceases after the pressure has been withdrawn is a classical
sign. Symptoms may be elicited when pressure is applied to an individual cusp
 Vitality tests: Vitality tests for individual teeth are usually positive
86. Most characteristic of irreversible pulpitis?
D. pain on cold,
E. Lingering pain on cold
F. pain in percussion
87. Definition of endodontic flare up?
A. Acute exacerbation of periradicular pathosis after start of RCT

21
Second lecture

1. During resection of the tip of the root in apicoectopmy, the incision should be:
A. Prependicular to ling axis of tooth
B. Parallel to long axis
C. Acute angle
D. Obtuse angle
2. What is the concept of Protaper system?
A. Step down technique
B. Step back technique
C. Crown down technique
D. Crown back technique
3. H File better than K files by:
A. Positive rake angle
B. Negative rake angle
C. No difference
D. Flexibility
4. The first step for removing gutta-percha is using a:
A. Solvent
B. New hedstorm file
C. Barbed Broach
D. Reamer
5. What is true about odontoblastic layer?
A. Filled with odontoblastic processes
B. Characterized by cell-to-cell gap junction
C. Characterized by cell-to-cell tight junction
D. Have higher concentration of cells
6. Which of the following is the most significant cause of ledge formation?
A. Infection

22
B. Remaining debris within the canal
C. No straight-line access
D. Improper obturation
7. What is the smallest area in root canal:
A. Apex in radiograph
B. Cementoenamel junction
C. Dentinoenamel junction
D. Cementodentinal junction
8. What is the advantage of AH plus over AH26?
A. Eugenol was added
B. Sealing ability is better
C. Formaldehyde release reduced
D. Cheaper
9. What is the Intracanal pressure is:
A. 10mm/Hg
B. 5mm/Hg
C. 20m/Hg
D. 55mm/Hg
10. When is the time to do re endo if the canal is exposed to oral fluids after root canal
treatment was performed?
A. 6 days
B. 2 weeks
C. 1 month
D. 2 months
11. Temperature at which Guttapercha melts:
A. 40-50
B. 50-60
C. 70-80
D. 90-120
12. A 30 year old man received trauma to his upper central incisor which caused root
fracture between middle and apical thirds. What is the first step to manage this
fracture?
A. Extract coronal part
B. Do RCT for entire tooth
C. Do RCT for coronal part
D. Splint both parts of fractured root
13. What type of splinting is done for horizontal root fracture?
A. A Flexible splinting
B. Fixed splinting
C. IMF
D. All of the above
23
14. Patient with pain the test reveals no response to percussion and no response to cold
test. There was severe pain when the dentist did a heat test. Electric pulp test response
for the tooth was at 18. The response for the control tooth are as follows : Percussion-
no response Cold test- response at 5 seconds Heat test- response at 5 seconds Electric
pulp test- response at 25 seconds. What is the pulpal diagnosis?
A. A Healthy pulp
B. Irreversible pulpitis
C. Necrotic
D. Reversible pulpitis
15. A patient reports to your clinic with continuous spontaneous pain, that kept her
awake in the night. On radiographic examination there was no radiographic changes.
There was prolonged pain on Heat testing. Diagnosis is:
A. Reversible pulpitis
B. Irreversible pulpits
C. Necrosis
D. Insufficient information
16. The bacterial flora of an infected previously treated endodontic tooth is best
characterized as:
A. Gram negative strict Anaerobes
B. Gram positive facultative Anaerobes
C. Gram negative facultative Aerobes
D. Gram positive strict aerobes
17. Which organism is mainly responsible for Endodontic failure:
A. Staphylococcus salivarius
B. Actinomyces and enterococcous faecalis
C. Streptococcus mutans
D. Pseudomonas aeruginosa
18. Cracked tooth syndrome is best diagnosed by?
A. Radiograph
B. Subjective symptoms
C. Palpation and vertical percussion
D. Pulp testing
19. The basic difference between K files and reamers is:
A. The number of spirals or flutes per unit length
B. The geometric cross Section
C. The depth of flutes
D. The direction of the spirals
20. A previous root canal treated patient visit your clinic. The RCT was done 5 years ago
but the treatment is poor and is a failure and a large periapical lesion. Your 1st choice
of treatment is:
A. Extraction
24
B. Apicectomy
C. Re-endodontic treatment
D. Implant
21. What is not commonly used in endodontic diagnosis
A. Ice
B. Electric pulp test
C. Hot water
D. Percussion
22. What is the flap used for apicectomy
A. Semilunar
B. Submarginal scalloped
C. Apical undisplaced
D. It does not make difference
23. What is the advantage of ultrasonic over conventional method in endodontics
A. Faster
B. More removal of debris
C. Better canal shaping
D. Less perforation
24. How do you differentiate between acute apical abscess and chronic apical abscess?
A. Vitality
B. Sinus tract
C. Percussion
D. Pain
25. For a tooth with bone and root resorption, where should the termination of RCT end?
A. 1 to 1.5 mm short of radiographic apex
B. 2 mm short of radiographic apex
C. At radiographic apex
D. At anatomic apex
26. Simplifill type of obturation is used in which rotary system
A. Protaper
B. Reciproc
C. Lightspeed
D. Revo
27. Symptomatic apical periodontitis compared to asymtomatic apical periodontitis
A. Former causes pain because it is dominated by Enterococcal feacalis
B. Later is dominated by Streptococcus
C. Former causes pain due to Peptostreptococcus species
D. Fungal infection in both
28. An endodontically treated maxillary second premolar, with moderate mesial and
distal caries is best restored by:
A. Amalgam
25
B. 3/4 crown
C. Full crown
D. Onlay
29. Most reliable pulp test
A. Electric tests
B. Thermal tests
C. Cavity test
D. Anethesia test
30. What is the diameter tip size of GG #1 according to ISO system :
A. 20
B. 30
C. 40
D. 50
31. To get cone size 24, the following length should be cut from cone size 20:
A. 1 mm
B. 2 mm
C. 3 mm
D. 4 mm
32. What is true regarding Gates Glidden
A. Made of nickel titanium
B. Numbered from 1 to 7
C. Can bind to walls of narrow canals
D. Flexible
33. Main disadvantage of Mcspadden technique?
A. Time consuming
B. Not used in straight canals
C. Not used in curved canals
D. All of above
34. We use Gates Glidden burs up to:
A. Pulp orifice
B. Coronal part of canal
C. Middle part of canal
D. Apical part of canal
35. Laser used in endodontics during root canal preparation is
A. CO2 Laser
B. ND (YAG) Laser
C. LED Laser
D. Argon laser
36. Lentulo-spiral is used during post and core placement to help with
A. Remove remaining gutta-percha
B. Cementation of post
26
C. To take impression of post space
D. To remove pulp tissue
37. A patient visits your clinic with a complaint in his tooth. IOPA of the tooth reveals
underobturated RCT. You begin to do a re-RCT, but the file does not go till the full
working length. There was no sticky spot was found. What is the reason?
A. Fracture instrument
B. Debris
C. Ledge
D. Perforation
38. The most superior cold test for pulp:
A. Ethyl chloride
B. Ice block
C. Carbon dioxide snow
D. Cold water
39. The correct access cavity preparation for the maxillary central and lateral incisor for
CT preparation is
A. Oval
B. Quadilateral
C. Round
D. Triangular
40. The access opening for the mandibular second molar
A. Triangle and base towards the mesial
B. Triangle and base towards the buccal
C. Triangle and base towards the distal
D. Trapezoidal
41. An abnormal response to heat thermal pulp test indicates:
A. Pulpal and periapical disorder
B. Vital pulp
C. Non vital pulp
D. None of the above
42. A patient comes to your dental clinic complaining of severe pain. There was no
response to pulp tests, but when you do percussion the patient jumps of pain. There is
no periapical change in the radiograph. What is the most likely diagnosis?
A. Acute apical abscess
B. Chronic apical abscess
C. Symptomatic apical periodontitis
D. Asymptomatic apical periodontitis
43. The post that has the least chances for tooth fracture
A. Casted post
B. Fiber post
C. Stainless steel
27
D. Metal post
44. Narrowest part of the root canal is
A. CEJ
B. Apical foramen
C. Radiographic apex
D. Apical constriction
45. During endodontic treatment a file broke in the root canal. If we can leave it in the
canal, which of the following options may have best prognosis
A. Long part away from working length at early stage of mechanical preparation
B. Short part near the working length at early stage of mechanical preparation
C. Long part near the working length at late stage of mechanical preparation
D. Short part away from working length at late stage of mechanical preparation
46. A patient returns to your clinic with pain on biting on his lower right second molar.
This tooth was treated endodontically 3 months ago. By looking at the radiograph, on
a bitewing, there is caries under restoration. On taking a periapical radiograph, there
is destruction of bone. What is your pulpal and periodontal diagnosis?
A. Previously treated and now with acute apical abscess
B. Previously treated and now with chronic apical abscess
C. Previously treated and now with acute apical periodontitis
D. Previously treated and now with chronic apical periodontitis
47. Which antibiotic is recommended to give for a patient with periapical abscess?
A. Penicillin
B. Erythromycin
C. Metronidazole
D. Tetracycline
48. Patient comes to your clinic with inadequate root canal treatment. You are planning
to do a re- root canal treatment. The tooth shows a periapical lesion, You will
obturate the tooth to
A. Obturate till superior end of lesion
B. 0.5 to 1 mm shorter of radiographic apex
C. 1 mm beyond radiographic apex
D. End of radiographic apex
49. The prognosis of the broken instrument in root canal is good when:
A. Instrument is less than 20 no.
B. Instrument is lodged in the bone beyond the apical area.
C. Instrument obliterates apical third of the canal.
D. Instrument is in middle 1/3 of root canal.
50. What is the minimal duration from when you carry out first RCT to then deciding to
carry out Re-endo treatment
A. 1 month
B. 2 weeks
28
C. Six days
D. 2 months
51. Post retention depends upon:
A. Post length
B. Post texture
C. Design of the post
D. All of the above
52. A patient visit your clinic with a complaint in the amalgam restoration done 2 years
before. Now the patient is having severe pain with localized swelling and pus. What
is to be done?
A. Abscess drainage
B. Complete debridement with abscess drainage
C. Antibiotics and analgesic
D. Wait for symptoms to subside and then endodontics
53. What is the type of Amalgam used for retrograde filling:
A. Zinc free
B. High copper
C. Mercury free
D. Antibiotic amalgam
54. Trauma to a 1 year old child's central incisor will affect the underlying permanent
tooth to cause:
A. Dens invaginatus
B. Dilaceration
C. Dens in dente
D. Eruption
55. A patient of 11 years of age is brought to the surgery by his parents after he suffered
trauma 30 mins ago. This caused avulsion of the upper central incisors. The tooth
were brought to the surgery in hand with 10 mins of dry time. Once the patient
reached the surgery, it was told to them that it should placed in milk, so then they did
it. At 11 years the central incisors have closed apex. What is your management ?
A. Replant and splint
B. Do RCT then replant
C. Replant and do RCT within 2 weeks
D. Doesn't need splinting
56. Patient came to you with trauma that caused the tooth to fall out of the socket 45
minutes ago. The avulsed tooth that was kept in cold milk what is the first
management?
A. Reimplantation
B. Put it in sodium sulfate
C. Put it in hanks balanced solution
D. Restoration

29
57. MTA is superior over other sealers because of
A. More biocompatibility
B. Higher tensile strength
C. Higher compressive strength
D. Superior sealing properties
58. A patient came to you after trauma to 4 incisor teeth and they do not respond to
electric pulp testing. IOPA X-ray of these teeth appears normal, what do you do?
A. Pulpotomy
B. Pulpectomy
C. Extraction
D. No need for treatment
59. Most common type of dental trauma in primary dentition
A. Avulsion
B. luxation
C. Extrusion
D. Fracture
60. After trauma, when tooth is partially displaced coronally injury is
A. Intrusion
B. Extrusion
C. Subluxation
D. Avulsion
61. Best media for keeping avulsed tooth
A. Water at room temperature
B. Milk at room temperature
C. Cold milk
D. Cold water
62. In which of the cases does trauma of a tooth cause PDL space widening?
A. Concussion
B. Subluxation
C. Luxation
D. Intrusion
63. A 10 year old male patient came to your clinic with carious exposure of a mature
tooth. What will your treatment be?
A. RCT
B. Pulp capping
C. Apexification
D. Pulpectomy
64. A child was brought to your office after tooth E fell out when he tripped and fell
playing outside. Mother says the tooth has been out for only 15 minutes and she put it
in milk right away and came to your office. The tooth looks viable and clean. You
should:
30
A. Reimplant the tooth immediately since it has been less than 1/2 hour
B. Clean the tooth with normal saline then reimplant the tooth since it has been less than
1/2 hour
C. Do not reimplant the tooth
D. Reimplant the tooth, perform a pulpotomy, splint, and follow-up in 1 week
65. Most important factor in avulsion for success of reimplantation:
A. Contamination of root
B. Time since avulsion
C. Tooth avulsed
D. Carry out ret whilst outside mouth
66. The greatest incidence of trauma to the permanent teeth occurs at
A. 8 to 12 years
B. 1.5 to 2.5 years
C. 15 years
D. 18 years
67. Maximum time for avulsed tooth to be re-implanted is
A. 1 hour
B. 24 hours
C. 8 to 9 hours
D. 1 to 2 weeks
68. Which one of the following options could result in referred pain?
A. Periapical abscess
B. Irreversible pulpitis
C. Reversible pulpitis
D. Periodontal abscess
69. Which of the following is the best radiographic technique to identify a suspected
horizontal root fracture in a maxillary anterior central incisor?
A. Multiple Water's projections
B. Multiple angulated periapical radiographs in addition to a normal, parallel-angulated,
periapical radiograph
C. A panoramic radiograph
D. A reverse Towne's projection
70. When you want to do a re-RCT and remove gutta-percha from the canal, what is the
first thing you use?
A. New hedstrom file
B. Chemical solvent
C. Heated plugger
D. Ultrasonics
71. When we use master cone 30 and cut 1mm from its tip, the diameter of tip is
A. 29
B. 30
31
C. 31
D. 32
72. The narrowest canal found in a three root maxillary first molar is the:
A. Mesio-buccal canal
B. Disto-buccal canal
C. Palatal canal
D. Disto-palatal canal
73. Best irrigant for endodontics
A. Sodium hypochlorite
B. Hydrogen peroxide
C. Saline
D. Sodium hydroxide
74. Cutting a molar into two halves and making them premolars is called
A. Apicectomy
B. Hemisection
C. Root resection
D. Bicuspidization
75. RCT treated patient returns to your dental clinic after 1 week with pain on RCT
completed tooth. What to do?
A. Re endo
B. Analgesic
C. Antibiotic
D. Apicectomy surgery
76. System-B obturation technique is?
A. Cold GP Condensation
B. Warm GP Condensation
C. Continuous condensation
D. Thermochemical condensation
77. Where is the spreader in endodontics used up to?
A. Beyond apex
B. 1mm short of radiographic apex
C. 1mm short of working length
D. Full working length
78. Master apical file (MAF) is:
A. File used to reciprocate
B. First file bind to working length
C. File to measure working length
D. Last file used to determine diameter of canal
79. In root of external resorption the pulp is
A. Reversible pulpitis
B. Irreversible pulpitis
32
C. Necrotic
D. Normal
80. Accessory canals are found:
A. Apical third
B. Middle third
C. Furcation area
D. Coronal third
81. On a radiograph, the facial root of a maxillary first premolar would appear distal to
the lingual root if the
A. Vertical angle of the cone was increased
B. Vertical angle of the cone was decreased
C. X-ray head was angled from a distal position relative to the premolar
D. X-ray head was angled from a mesial position relative to the premolar
82. The radiographic criteria used for evaluating the success of endodontic therapy:
A. Reduction of the size of the periapical lesion.
B. No response to percussion and palpation test.
C. Extension of the sealer cement through lateral canals.
D. None of the above.
83. What is the test to differentiate between chronic apical abscess and periodontal
abscess
A. Percussion test
B. Pulp test
C. Probing
D. Radiograph
84. What is used to locate orifice of canals?
A. Endodontic spreader
B. Barite probe
C. Endo file with curved tip
D. Round bur
85. Which of the following is the best aid in the diagnosis of an irreversible pulpitis?
A. Electric pulp test
B. Thermal
C. Percussion
D. Test cavity
86. Which one is histologically, the zone of pulp:
A. Cell rich zone contains fibroblasts
B. Cell free zone contains capillaries and nerve networks
C. Odonotblastic layer contains odontoblasts
D. All of the above
87. C shaoed canals are commonly seen in
A. Maxillary first molar
33
B. Mandibular second molar
C. Mandibular first molar
D. Maxillary second molar
88. Patient with esthetic concern in their central incisor of maxilla comes to your clinic. It
appears as though he has a dark disoclouration. Patient reports history of trauma 1
year ago. What is the treatment?
A. Crown
B. Veneer
C. Endodontic treatment
D. Endodontic treatment with internal bleaching
89. An elderly patient of 62 years came to your dental clinic complaining of discomfort
during biting. He has temporary RCT in his lower second molar and an old amalgam
restoration in opposing tooth that was done 15 years ago. There is deep pocket related
to 47 (5-7 mm) buccally and lingually with no evidence of periodontal disease in the
neighboring teeth. What is the likely cause of pain?
A. Apical periodontitis
B. Vertical root fracture
C. Over extending gutta percha
D. Micro leakage in 46
90. During instrumentation of an upper first premolar, you feel a sudden disappearance of
main root canal. This is most likely due to
A. Debris
B. Bifurcation of main canal
C. Apical perforation
D. Hypertrophic calcification in apical part
91. After you did an RCT for your patient, he came back to the clinic after few days with
severe pain on biting. You take an ×-ray and it revealed that the RCT filling is very
good, but you saw a radiopaque, thin (film like) spot on the lateral border of the root
what is the most probable diagnosis?
A. Accessory canal
B. Vertical root canal fracture
C. Ledge
D. Perforation
92. Electric pulp test affect which part of the pulp
A. Blood supply of pulp
B. Neural fibres of the pulp
C. Cells of pulp
D. All of above
93. Which root has extra Canal in maxillary first molar
A. Palatal
B. Distal
34
C. mesiolingual
D. mesiobucaal
94. What is the best method to detect working length for endodontics/RCT?
A. Precurve initial file before canal insertion
B. Take periapical radiographs using paralleling technique
C. Measures by calculating mm
D. Push small file passing the foramen
95. Which of the following teeth have bifurcated root most probably?
A. Maxillary canine
B. Mandibular canine
C. Mandibular lateral incisor
D. Maxillary lateral incisor
96. Which of the following diagnostic criteria is least reliable in the assessment of the
pulpal status of the primary dentition?
A. Swelling
B. Electric pulp test
C. Spontaneous pain
D. Internal resorption
97. Cells responsible for Periapical granuloma is
A. Neutrophils
B. Plasma cells
C. Macrophages
D. Leucocytes
98. An upper second premolar is treated by temporary RCT, 6 months ago as shown in
the figure. Patient has reported to your clinic with pain, but normal periodontal depth.
What is the reason of the pain?

A. Vertical root fracture


B. Missed lateral canal
C. Missed root canal

35
D. Microleakage
99. A patient presents in your dental clinic with an asymptomatic tooth with a necrotic
pulp, a broken lamina-dura in radiograph, and circumscribed radiolucency. The
periradicular diagnosis:
A. Acute apical periodontitis
B. Chronic apical periodontitis
C. Acute exacerbation of chronic apical periodontitis
D. Chronic apical abscess
100. Barbed Broach is used for
A. Widening of orifice
B. Extirpation of the pulp
C. Lateral canals
D. Used up to working length
101. We use a Mater Apical file to shape till the apex. Then we use larger files at shorter
length until we reach coronal end. What is the technique called?
A. Crown down
B. Vertical compaction
C. Step back technique
D. Step down technique
102. Sequence of endodontic instrumentation
A. Broach- Reamer-File
B. Reamer- File-Broach
C. File-Broach-Reamer
D. File-Reamer-Broach
103. Irreversible pulpits has pulp?
A. Vital
B. Vital but abnormal
C. Non-vital
D. Necrotic
104. A patient is in your surgery with a complaint of pain with sweet and cold. You take a
radiograph as below. What is your diagnosis?
A. Irreversible pulpitis
B. Reversible pulpits
C. Acute apical periodontitis
D. Chronic apical abscess
105. Pain on cold and on percussion
A. Irreversible pulpitis
B. Reversible pulpitis
C. Pulpal hyperemia
D. Apical periodontitis
106. Smallest no. file is
36
A. 4
B. 6
C. 8
D. 10
107. Patient reports to clinic with pain on tooth 15. You decide to do an RCT following
appropriate pulp testing. The patient complete RCT, but still has pain on percussion,
the next day. What is the diagnosis?
A. Primary apical periodontitis
B. Secondary apical periodontitis
C. Over instrumentation
D. Over medication
108. Best technique to remove inadequately condensed GP?
A. Automated instrumentation
B. Hand instrument
C. Ultrasonic
D. Rotary
109. Which of the below non-odontogenic lesion looks similar to an endodontic lesion
A. Dentigerous cyst
B. Initial stage of cemental dysplasia
C. Ossifying fibroma
D. Hyperparathyroidsm
110. NaOCI is used in RCT to
A. Removes smear layer
B. Softening the canal
C. Kills microorganisms
D. Disinfect the canal
111. All are contraindications for endodontic treatment except
A. Non-strategic tooth
B. Non restorable teeth
C. Vertical root fracture
D. Tooth with large periapical lesion
112. A patient comes to your clinic with a large carious lesion in the mandibular first
molar. The tooth has chronic pulpitis, but tests positive to vital test. On radiograph,
there is a large well defined radio-opacity in the periapical area. What is the
diagnosis?
A. Condensing osteitis
B. Cemental dysplasia
C. Periapical granuloma
D. Hypercementosis
113. Highest failure after endodontic treatment is seen in
A. First premolar
37
B. Maxillary first molar
C. Mandibular lateral incisor
D. Second premolar
114. What best describes post-operative pain?
A. It's called condensing osteits, if it continues for more than 3 days
B. Can be controlled by analgesics
C. Should subside after 1 day
D. Can be controlled by antibiotics
115. After root fracture, the best type of tissue healing is
A. Interposition of calcified Tissue
B. Connective tissue
C. Bone and connective tissue
D. Granulation tissue
116. The best prognosis of perforation after application of MTA is in the case of
A. Small Perforation below alveolar crest
B. Large perforation below alveolar crest
C. Small Perforation at level of alveolar crest
D. Large perforation at level of alveolar crest
117. Root perforation is treated by
A. MTA
B. Ca (OH) 2
C. Root canal with GP
D. Tetracycline
118. The normal response of a vital pulp to the electric pulp testing is
A. No response
B. Higher than that of control tooth
C. Lower
D. In a range similar to that of control tooth
119. Patient has dull pain related to lower molar tooth which was endodontically treated 3
years ago as shown in the image below. What is the likely reason for failure?

38
A. Missed canal
B. Vertical root fracture
C. Short endodontic BMP and inadequate filling
D. Coronal leakage
120. How can you differentiate between endodontic and non-endodontic pathosis
A. Radiographic appearance
B. Radiographic location
C. Pulp sensibility testing
D. Percussion test
121. A patient gives history of previous intrusion of incisors. IPA shows loss of
periodontal ligament space and progressive root resorption. What is your conclusion?
A. Inflammatory resorption
B. Replacement resorption
C. Surface resorption
D. Cervical root surface resorption
122. Successful endodontics using rotary techniques is achieved by
A. Use of EDTA
B. Pre-enlarged coronal flare
C. Proper irrigation
D. Good obturation
123. EDTA is primarily used to
A. Dissolve organic matter
B. Dissolve inorganic matter
C. Kill bacteria
D. Prevent sealer from extruding out of the canal space
124. In the treatment of an acute periodontal abscess, the most important first step is to
A. Prescribe systemic antibiotics
B. Reflect a periodontal flap surgery
C. Obtain drainage
D. Prescribe hot salt mouth washes
125. Removal of old gutta percha from canal is best done by
A. Chemical
B. Mechanical
C. Thermal
D. Combined heat and chemical methods
126. When you want to instrument a curved root, you insert a file by:
A. Putting gauze on the file & bend it by hand
B. Bend the file by pliers
C. By bare finger
D. By twist
127. Proteolytic irrigant used in root canal treatment is
39
A. Water
B. Sodium hypochlorite
C. Saline
D. EDTA
128. Material used for managing weeping canal
A. Glass ionomer cement
B. Compomer
C. Calcium hydroxide
D. Sodium hypochloride
129. Rarefaction of bone diagnosis is by:
A. pain from the cold
B. pain by percussion.
C. cortical bone resorption
D. radicular bone resorption
130. Constituent of AH26
A. Formacresol
B. Zinc oxide
C. Epoxy Resin
D. Xylene
131. Main content in gutta percha obturating material
A. Zinc oxide
B. Gutta percha
C. Choloroform
D. Zinc acetate
132. The most common cause of discoloration of a single tooth
A. Endodontic treatment
B. Tetracycline stain
C. Dentiogenesis imperfecta
D. Amelogenesis imperfecta
133. Which is always associated with a non-vital tooth
A. Chronic apical periodontitis
B. Internal resorption
C. Hyperplastic pulptitis
D. Condensing ostetitis
134. Identify the below lesion

40
A. Nasopalatine cyst
B. Nasolabial cyst
C. Periapical cyst
D. Residual cyst
135. A 2% taper endodontic file means:
A. Distance of the file from DO to D16
B. 0.02 mm is the diameter at tip, increase is from the tip to the handle
C. 0.2 mm is the diameter at tip, increase is from the tip to D16
D. 0.02 mm increase in diameter per 1 mm increase in length
136. Apical foramen located
A. Lateral to apex
B. At apex
C. At lateral canal
D. At accessory canal
137. Size of 20 file at tip is
A. 20 mm
B. 2 mm
C. 0.02 mm
D. 0.2 mm
138. There is a horizontal root fracture in the middle third of the root of central incisor in
an 11-year- old patient. The tooth is mobile and vital. Vitality tests show no response.
How should this be treated?
A. Extract
B. Pulpectomy immediately and splint
C. Splint and observe

41
D. Do nothing and follow-up in 10 to 14 days
139. The most suitable description to describe NiTi file is
A. Rigidity
B. Low friction
C. Stainless
D. Hard
140. Cause of Ni-Ti breakage in endodontic
A. Cyclic fatigue
B. Torsional stress
C. Rigidity
D. A and B
141. Material used in the majority of areas of perforation
A. Mineral Trioxide Aggregate
B. Gutta percha
C. CaOH
D. GIC
142. Continuous condensation technique for GP filling is
A. Obtura I
B. Obtura Il
C. Ultrafill
D. System B
143. Stieglitz pliers is:
A. Used to contour matrix bands
B. Used to hold the silver point
C. Used to remove roots during hemisection
D. Used to place rubber dam clamp on the tooth
144. Best method to protect the teeth after bicuspidization is
A. Composite restoration
B. Amalgam restoration
C. Crown
D. Onlay
145. Minimum length of Gutta Percha that is required to be left when preparing for a post
preparation to avoid leakage and reinfection
A. 3-4mm
B. 4-6mm
C. 6-8 mm
D. Post length is not important
146. The main link between the pulp and the periodontium is
A. Apical foramen
B. Dentinal tubules
C. Accessory canals
42
D. PDL
147. Access opening of maxillary lateral incisor
A. Triangular
B. Oval
C. Trapezoidal
D. Round
148. You want to remove gutta percha to prepare post space. What is the best method to
maintain obturation integrity and reducing perforation risk?
A. Mechanical drill
B. Chemical solvent
C. Heated plugger
D. Ultrasonic
149. Patency file is
A. File that reach apical constriction
B. File that you use with irrigation for recapitulation
C. Recapitulation with bleach
D. Rotate the file circumferentially at the walls to remove any block of lateral canals
150. Within the root of internal resorption, the pulp shows
A. Reversible pulpitis
B. Irreversible pulpitis
C. Necrosis
D. Normal
151. Endodontic instrument standardization is based on
A. Height
B. Length
C. Width of tip
D. Handle
152. The radiographic criteria used for evaluating the success of endodontic therapy
A. Reduction of the size of the periapical lesion
B. No response to percussion and palpation test
C. Extension of the sealer cement through lateral canals
D. None of the above
153. A patient presents to your clinic with pain on biting (relief when open) related to
tooth with big amalgam restoration. Thermals pulp testing results were normal, so
was the xray. What is the best method to confirm the diagnosis?
A. Periapical X ray
B. Electric pulp test
C. Tooth sloth
D. Percussion
154. If a maxillary first molar is affected by periodontal disease it is expected that the
furcation which will be involved first is the:
43
A. Distal furcation
B. Buccal furcation
C. Mesial furcation
D. All furcation have the same possibility
155. A patient came complaining of severe pain on biting, related to a certain tooth. Upon
examination no pulpal or periodontal findings were found, and pulpal vitality is
positive, and there was no pain on percussion your diagnosis is
A. Complete root fracture
B. Partial fracture with pulp involvement
C. Partial fracture without pulp involvement
D. Irreversible pulpitis
156. The Ideal form for the wall of root during RCT preparation is
A. convergent toward occlusal surface
B. flaring toward the occlusal surface
C. parallel toward occlusal surface
D. Angled toward occlusal surface
157. Active fibers in necrotic pulp
A. A fibers
B. B fibers
C. C fibers
D. No fibers
158. A patient of 48 years comes to clinic with complain of missing tooth. He is ideally
looking for a FPD for a missing tooth as he cannot afford implant. You want to find
the right abutment for the FPD. After examination you find that the abutment you
chose will need RCT first because the pulp is stressed. What is the meaning of
stressed pulp?
A. Pulp exposure with severe pain
B. Non restorable tooth
C. Tooth has large old restorations
D. Broken tooth
159. A patient of 34 years and male comes to your clinic with complaint of brief sharp
pain. A tooth cusp is broken and dentine is exposed. What is the pulpal status?
A. Reversible pulpitis
B. Irreversible pulpitis
C. Activated A fibers
D. Normal pulp
160. Endodontic files are of lengths
A. 21, 23, 25 mm
B. 21, 25, 31 mm
C. 21, 31, 41 mm
D. 21, 27, 31 mm
44
161. A 25 year old patient present your clinic complaining of discomfort in his mandibular
second premolar. After taking an x-ray, you view it under good lighting to note that
there's a sudden dark disappearance of the canal on midroot. Most probable
explanation is:
A. Hypertrophic calcification in apical part
B. Secondary dentin apically
C. Bifurcation area
D. Polyp
162. A 22 year-old student reported to the Emergency Clinic complaining of severe pain
when chewing food on the lower first left molar. Radiographically, there are no caries
and the periapical area is normal. The clinical examination reveals good oral hygiene
and a positive response to thermal test, electric pulp tester, and percussion test. The
patient said that a dentist cemented a bridge on the upper right molar area three days
ago. What is the most likely diagnosis?
A. Necrotic pulp
B. Chronic apical abscess
C. Acute apical periodontitis
D. Irreversible pulpitis
163. A patient came to your clinic with pain in his mouth. On trying to find the source of
the pain through history, it was apparent that the patient was unable to locate the pain.
You decide to do an anesthesia test. Which area will you start the test?
A. Mandibular anteriors first
B. Maxillary molars first then anteriors
C. Maxillary premolars
D. Mandibular canines
164. Stressed pulp means
A. Asymptomatic tooth with numerous restorations in several years
B. Tooth not responsive to pulp test
C. Tooth with no restoration
D. Tooth with recent restoration and metal crown
165. RCT abutment of FPD has higher risk for fracture. Abutment which has RCT in a
cantilever FPD has higher susceptibility to fracture.
A. First statement is true, second is false
B. First statement is false, second is true
C. Both are false
D. Both are true
166. Which of the following is contraindication for RCT?
A. A Vertical tooth fracture
B. Horizontal root fracture
C. Half crown fracture
D. Periodontitis
45
167. What does removal of smear layer in the root canal system?
A. Bacteria allowed to enter dentine tubules
B. Makes perforation
C. Better for shaping and obturation
D. Alters permeability of the dentine
168. What is MTA?
A. Mineral Tymol Aggregate
B. Mineral Trioxide Aggregate
C. Mineral Trioxide Acetate
D. Metal Trioxide Aggregate
169. The proper treatment for an avulsed tooth that occurred less than 25 minutes include
which of the following? ago would
A. Minimize contact with root and replant primary teeth
B. Placement of a rigid splint for 4 to 6 weeks
C. Replant and splint for 7 to 10 days
D. Immediate pulpectomy and reimplantation
170. A patient of 7 years comes to your clinic from dental trauma. The avulsed permanent
tooth was brought to the dentist with all appropriate media and storage, which was
then carefully reimplanted within 20 minutes. Splinting should ideal be done for
A. One to two weeks
B. Two to three weeks
C. Three to fours weeks
D. Four to eight weeks
171. A patient is in your clinic with an Avulsed central incisor that happened 90 minutes
ago. How long is this tooth recommended to be splinted for?
A. 1 week
B. 2 weeks
C. 3 weeks
D. 4 weeks
172. Child suffers trauma to his lower four incisors and leaves them without any
consultation or treatment for four years. He now has pain and swelling. On
radiogrpahic examination there appears to be radiolucency below four incisors. What
should be done?
A. RCT for all teeth and root planing
B. Vitality test to teeth and RCT to be done on non-vital teeth.
C. Extract teeth
D. Wait for resolution
173. A 14-year-old boy reports to your clinic after trauma to his tooth. His anterior
mandibular central incisor has been avulsed. It has been for 45 minutes. What is the
management?
A. Pulpectomy with CaOH then reimplant
46
B. Reimplant and splint and B pulpectomy with CaOH after a week
C. Reimplant and splint and observation for 3 weeks
D. Implant
174. An intrusive injury to a primary tooth of a 2 year old will cause
A. Hutchinson's incisor
B. Pulpitis
C. Enamel hypoplasia
D. Resorption
175. An endodontically treated tooth was restored with a cast post-and-core and a metal-
ceramic crown Three months later, the patient complains of pain, especially on biting.
Radiographic findings an tooth mobility tests are normal. The most probable cause of
pain is
A. A loose crown
B. Psychosomatic
C. A vertical root fractures
D. A premature eccentric contact

176. Ah plus setting time ?


A. 6 hrs
B. 8 hrs
C. 9 hrs
 AH- Plus: 4 hours working time, 8 hours for setting, and don’t release formaldehyde.
 AH- 26: 4-6 hours working time, 9-15 hours for setting, releases formaldehyde.
177. Best storage medium for avulsed teeth ?
A. HBSS
 From best to worse :
1. Cell culture media HBSS.
2. Low fat cold milk.
3. Physiological saline.
4. Saliva.
5. Water .
178. Apexification and Apexogenesis definition? ?
 Apexification : Pulpectomy of immature permanent tooth
 Apexogenesis : Pulptomy of immature permanent tooth.
179. Best prognosis in teeth ?
A. Vertical bone loss
B. Horizontal bone loss

47
180. Only teeth show 8 configurations?
A. Upper 4
B. Upper 5 ( Max 2nd premolar )
C. Upper 6
D. Lower 6

181. Which one removes the smear layer ?


A. ETDA
B. Sodium hypochlorite
C. Chlorhexidine
182. Best location & size of root perforation that is favorable?
A. Small perforation below height of bone crest
B. Large perforation below height of bone crest
C. Small perforation at height of bone crest
D. Large perforation at height of bone crest
 Perforation near cervical → Poor prognosis.
 Perforation near apical → Good prognosis.
 Perforation subcrestal→ Bad prognosis.
 Perforation supracrestal→ Good prognosis

48
183. Pt has caries, on periapical radiograph revealed radiopacity under apices ?
A. Periapical granuloma
B. Condensing osteitis
184. Cause of a file broken in x-ray ?
A. improper access line
185. Best way to apply sealer into the canal?
A. Files
B. Peeso reamer
C. Lentulo spiral
 Lentulo spiral instrument used to spread sealer in the canal
186. Caries continues to root and the patient needs RCT treatment. What does the doctor do
before starting the treatment?
A. Check on restorability
B. Start RCT
187. What does a localized periodontal defect around an endodontically treated tooth suggest?
A. Gingivitis
B. Internal resorption
C. External resorption
D. Vertical root fracture
 J-shaped radiolucency around endodontically treated tooth =
VRF
188. 60 years old patient presented with intermittent pain in tooth #26 that is associated with
sinus tract upon examination, tooth #26 is sensitive to percussion and palpation,7mm
probing depth in the mesiopalatal area of the tooth and grade II mobility. Which of the
following is the most probable diagnosis of tooth #26?
A. Previously treated with acute apical abscess
B. Previously treated associated with vertical root fracture
C. Previously treated associated without vertical root fracture
D. Previously treated with asymptomatic apical periodontitis
189. Patient have bone lose in lower 6 it's about 7mm in mesial side, so the diagnosis is ?
A. Hemi section
B. Vertical fracture
C. RCT
D. Extraction
190. A 22 - year-old man presented to the clinic complaining of shooting pain on biting his
maxillary left tooth, which subsided as soon as he released the bite. On clinical
examination, tooth # 25 had a large disto-occlusal composite filling, and was tender to
both vertical and horizontal percussion. The patient mentioned that he was dieting and
frequently ate nuts. The gingival tissue around the tooth was healthy. There were other
teeth in his mouth that required simple filling (see report) Radiographic examination:
showed that the tooth was root canal treated and the restoration margins were intact.
Which of the following is the most likely diagnosis of this tooth ?
A. Cracked tooth
B. Root resorption
C. Periapical abscess
D. Periapical periodontitis

49
191. A 22-year-old patient has upper #12 fractures with pinhole pulp exposure 12? before 1
hour, what is the Tx?
A. RCT (more than 1 hour)
B. Direct pulp capping
C. Pulpotomy
 Note: According to the time we will decide, if less than 1 h= DPC (Direct Pulp
Capping) or pulpotomy + Depending on size of pinhole < 0.5
192. Patient came to do RCT, after cleaning and shaping there is drainage, how to manage ?
A. keep it open
B. put temporary restoration
C. put composite
D. do perforation
193. Crown fracture involving enamel, dentin and exposing the pulp, classify?
A. Crown infarction
B. complicated crown fracture
C. uncomplicated crown fracture
 Uncomplicated involving enamel +dentin
 Complicated involving enamel +dentin +pulp
194. what is your diagnosis for this picture

A. internal resorption
B. external resorption
195. location of second canal in lower canine ?
A. Mesial
B. Distal
C. Labial
D. Lingual
196. In the mid of root suddenly the canal disappears means?
A. Second canal (in x-ray)
B. Internal resorption

C. calcified canal (during working)


 Could be either classified or ( ledge , transportation ) In case the 2 canals join very
common mb1 and mb2 when u irritated one of canals the 2end canal showed back
flow of irrigation
 If the scenario canal disappears in the X-ray so usually 2end canal occur

50
197. Dentist doing RCT, suddenly bleeding occurred through the canal. The dentist noticed
that the rubber stopper on the file has moved from it's original location? What is
happened?
A. Internal resorption
B. External resorption
C. Perforation
D. Ledge
198. Pt came on a routine checkup. Dentist noticed an overextended cone 5 mm on #14.
Patient had his RCT 10 years ago. Asymptomatic no signs of peri apical pathosis on x-
ray. Management?
A. Follow up.
B. Surgical removal of the extended cone
199. Case endo used step back technique with file 90 ?
A. 0.9 tapered 0.4
B. 0.9 tapered 0.06
C. 0.9 iso standard 0.4
D. 0.9 iso standard 0.6
200. A 30-year-old patient complains of severe pain in #36, x-rays were performed and it was
found that the canal curved. What is the appropriate system?
A. rotary
B. hand instrument only
C. both
D. avoid Gates glidden
201. Which tooth constant number of canal ?
A. Maxillary central incisor
B. Mandibular central incisor
C. Maxillary canine
D. Mandibular canine
202. Which of the following canals in(6) is most difficult to locate?
A. palatal.
B. Distobuccal.
C. Mesiobuccal.
D. All of above
203. 30-year-old patient has sinus drain and Not responding to cold test with slight tender to
percussion What is the diagnosis?
A. irreversible pulpitis normal apical issue
B. irreversible pulpitis chronic apical abscess
C. necrotic pulpitis asymptomatic periodontitis
D. Necrosis pulp with chronic apical tissue
 Sinus drain = chronic.
 No response to cold (pulp test) = necrotic.
 Pain on percussion = periapical issue or periodontitis
204. pt with swelling in the face started before 2 days ago, has a tooth with no response to cold
and tender to percussion in radiograph no radiolucency diagnosis ?
A. acute abscess
B. chronic abscess

51
C. necrotic with symptomatic periodontitis
D. necrotic with asymptomatic periodontitis
 the correct answer showed be necrotic with acute apical periodontitis
205. what is the most cause necrosis?
A. Avulsion
B. Extrusion
C. subluxation
D. concussion
 between the options…. (Concussion least, Most= intrusion if in the options)
206. Intrinsic stains after RCT due to?
A. Trauma to dentinal tubules
B. blood from dentinal tubules
C. restoration remnant
207. cutting of endo file?
A. 16
208. Carrier based technique ?
A. Obtera 3
B. Thermafil
 Thermomechanical compaction = McSpadden Compactor
 thermoplastic injection technique = Obtura II (internal resorption)
 thermoplasticized technique open apex (external resorption)
 warm vertical compaction + continuous wave compaction technique =System B
 Carrier-Based Gutta-Percha = thermafil + pro taper + SuccessFil case
209. System-B obturation technique is?
A. Cold GP. Condensation
B. Warm GP. Condensation
C. Continuous condensation
210. Thermal compaction technique is?
A. therma fill
B. Obtura II
C. McSpadden
211. Internal resorption?
A. Thermoplastic injection
B. Thermoplasticized
212. Broken file and mange by doing bypass what should do ?
A. Tell the pt
B. Don't tell him and continue ttt
213. Definition of concussion trauma?
A. An injury to the tooth-supporting structures without increased mobility or
displacement of the tooth, but with pain to percussion
B. An injury to the tooth-supporting structures resulting in increased mobility, without
displacement of the tooth
C. Partial displacement of the tooth out of its socket, in occlusal/coronal direction
D. A displacement of the tooth in a direction other than axially
214. Def of Subluxation?
A. loosing but no displacement of the tooth

52
215. Def of Concussion ?
A. no loosing and displacement of the tooth
216. Splinting time for alveolar fracture ?
A. 2 weeks
B. 3weeks
C. 4 weeks
 Splinting 4 weeks: for 1-Lateral luxation 2-Mid root fracture 3-Alveolar fracture 4-
Avulsion (delayed replantation)
217. Splinting time of avulsed tooth Less than 1 hour?
A. 2 weeks
B. 3 weeks
C. 4 weeks
218. Master apical cone extruded, how to manage ?
A. re instrument
B. choose larger cone
C. chloroform solvent
219. Dentist performed RCT using AH26 sealer and the sealer was extruded outside the apical
foramen your management?
A. follow up
B. apicoectomy
 if say there are symptoms, choose B
220. MTA drawback?
 Discoloration (main)
 presence of toxic elements in the material composition(main)
 difficult handling
 long setting time
 high material cost
 absence of a known solvent for this material
 difficulty of its removal after curing
221. How is a vertical root fracture best confirmed?
A. Bite test
B. Percussion
C. Exploratory surgery
 Best radiograph for diagnosis of root fracture cbct.
 Best method to diagnose root fracture: exploratory surgery
222. Which of the following is a reciprocating endodontic files system?
A. Endo-Eze
B. EndoSequence
C. ProFile
D. Vortex
 Endo-Eze, WaveOne, Reciproc Reciprocation systems.
 EndoSequence, ProFile Vortex, ProTaper Universal, K3, ProFile Rotary systems
223. Best material to repair Perforation in furcation ?
A. GIC
B. Zinc oxide eugenol
C. Calcium hydroxide

53
D. Bio-ceramics
224. Where is the narrowest diameter in root?
A. Anatomical apox
B. Radiographic apex
C. Physiological apex
D. apical constriction
225. The pt have PFM crown and he needs RCT but he doesn't want to remove the crown, the
access cavity will be done by ?
A. Carbide bur
B. Dimond bur
C. Diamond for poreclin and transmetal bur for metal
D. Transmetal for poreclin and diamond for metal
226. Management of internal resorption?
A. apply caoh for 1 week then obturation.
B. Obturate immediately
C. apply caoh for 2 weeks then obturation
227. Pulp exposure in trauma for 3 days?
A. Pulpectomy
B. Pulp capping
C. Cervical pulpotomy.
D. Pulpotomy
228. Negative rake angle in ?
A. K-files
B. H- files
 K file -ve rake angle. H file +ve rake angle
229. The length of working part in file 20 is?
A. 12
B. 16
C. 21
 No matter how the file changes, the length of the working part will be 16
230. Management of NaOCI accident?
A. anathesia and cold compression to induce vasoconstriction and control bleeding -
irrigation with normal saline-2nd day warm compression.
Which of the following is the endodontic emergency treatment for teeth with symptomatic
irreversible pulpitis ?
A. Pulpectomy
B. trephination
C. direct pulp capping
D. incision and drainage
231. Which of the following is an error occurs during obturation of the root canal ?
A. voids
B. ledge formation
C. broken instrument
D. filling lateral canal
232. What is the use of root canal Sealer in endodontics?
A. Fill the voids

54
B. Increase strength of RC filling
C. Disinfect the canal
D. Irrigation of canal
233. What does a localized periodontal defect around an endodontically treated tooth suggest?
A. gingivitis
B. internal resorption
C. external resorption
D. vertical root fracture
234. percentage of gutta percha in composition?
A. 100% gutta percha
B. 50%gutta percha 50% wax
C. 70% gutta percha 30% zinc oxide
D. 20%gutta percha 80%zinc oxide
235. Fracture file in side the canal and you used the ultra sonic tip to make space around
broken file what the other instrument you can use to retrieve the broken file?
A. Eggler
B. Instrument kit remover
C. Stiglitz forcep
 Eggler =post remover
 Instrument kit remover=retrieve file
 Stiglitz forcep = remove silver point
236. Chid have trauma for his teeth and come after 3 hours and have pulp exposure What do u
do?
A. rct
B. pulpectomy
C. dircet pulp capping
237. Child with trauma avulsed teeth come in less than an hour.
A. Reimplant with rigid fixation
B. Reimplant with nonrigid fixation
C. no need for reimplant it will fail (if primary tooth )
238. after a minor accident, the 5 years Child have tooth which have no mobility but have
intrusive luxation, best treatment ?
A. acrylic splint
B. non rigid fixation
C. rigid fixation
D. extraction
 The best treatment is extraction so that there is no injury to permanent germ,
 If permenant tooth :
 open apex= allow to reerupt
 closed apex reposition Then RCT
 if less than 3mm= observation
 If more than 7mm= surgically reposition splint 4-8 weeks
239. Failure of pulpotomy lead to?
A. Internal resorption
B. External resorption
240. Root Resorption inhibition property after replantation of avulsed teeth?

55
A. Edta
B. Ledermix
C. Sodium perborate
241. Material used for Apexification?
A. Caoh
B. mta
242. Conditioning of resected root end by ?
A. Citric acid.
B. EDTA!!!!
C. Phosphoric acid.
D. H202
243. Weeping canal we use ?
A. Gutta percha.
B. Calcim Hydroxide
 What is weeping canal ? Canal from which
constant clear or reddish exudation is
appeared
244. during cavity preparation u cause perforation,
what sould do ?
A. put mta then continue
B. continue and put mta at the last
245. Resorption of zinc oxide eugenol in primary teeth ?
A. Slower
B. Rapid Bio
 Root Resorption first then ZOE resorption.
246. best storage medium for avulsed teeth ?
A. Hbss
 from best to worst media: 1-cell culture media HBSS 2-low fat cold milk 3-
physiological saline. 4-Saliva 5-water
247. Healthy pt came for RCT, you administered IANB and long Buccal infiltration, while
opening cavity at rated to feel pain and you injected another IANB, upon approaching
pulp he suffered from pain, what’s the next step?
A. Intraligamentary injection
B. Second long Buccal infiltration
C. Third IANB
D. Dismiss the patient and prescribe antibiotics
248. Coronal Lateral perforation observed with no. 15 K file small in size and supracrustal
location, managed with MTA, what's the prognosis?
A. Good
B. Bad
 Supracrestal perforation = good prognosis
 Subcrestal perforation = bad prognosis
249. Method to assess blood flow in pulp?
A. Laser oximetry
B. doppler flowmetry
 Pulse oximetry measure= oxygen concentration in blood

56
 Doppler flowmetry = assess blood flow
250. Instrument fracture at middle part, management ?
A. Bypass
B. Retrieve it
 Instrument fracture and management:
 Apical = RCT and follow up
 Middle =bypass
 Coronal =retrieve it
251. Standard file taper ?
A. 0.02
B. 0.04
 Standard file taper :0.02
 ProTaper file taper :0.04, 0.06
 Diameter = file size /100
252. what the instrument for locate canal orifice?
A. Explorer
B. Scaler
253. Yellowish discoloration of tooth after RCT due to?
A. Hemorrhage from pulp
B. Incomplete debridement of pulp tissuev
C. Leaving Gutta percha in pulp chamber
D. Hemorrhage from canal + leaving Gutta percha
254. how to manage calcified canal
A. cbct , 1%methylene blue (stain dentin) and Champagne bubble test (naoh)
255. Reliable test for perforation?
A. Electronic apex locator
B. PA
C. Paper points
D. Apex locator
256. 30-year-old patient has sinus drain and Not responding to cold test with slight tender to
percussion What is the diagnosis?
A. irreversible pulpitis; normal apical tissue
B. irreversible pulpitis; chronic apical abscess
C. necrotic pulp; asymptomatic periodontitis
D. Necrosis pulp with chronic apical tissue
257. To mimize rotary file fracture
A. increase pressure on file
B. light adaptation with increase water coolant in rotary
C. coarse grain and increase speed
D. increase torque or something like that
 The most important thing in the rotary file is so that the file does not break, so we
press hard + be in EDTA + cooling
258. You have a pt came to continue his initiated pulp therapy by another dentist, you inserted
the apex located file and immediately made the sound, what’s the reason?
A. You went beyond apex
B. There's Coronal perforation

57
259. Diameter of NITI file No. 30 with 0.04 taper at D9 ?
A. 0.66
B. 0.1
260. Bacteria in failed endo
A. e.feacials
261. Most characteristic of irreversible pulpitis?
A. pain on cold,
B. Lingering pain on cold
C. pain in percussion
262. Which of the following is the material used for coronal perforation seal?
A. Gutta-Percha
B. BioCeramic
C. Caoh
D. ZOE
263. Danger zone is?
A. Distal wall of MD root of the lower molar
B. Distal wall of MB root of lower molar

264. Patency filling?


A. Push the file apically to remove any block at the apex 
B. Rotate the file circumferentially at the walls to remove any block of lateral canals.
C. Rotary files circumferentially at the walls to remove any block of lateral canals.
D. File with bleaching agent.
 “A patency file is a small K-file (usually a size #10 or #15) that
 is passively extended slightly beyond the apical foramen. The use of a patency file has
been suggested for most rotary techniques. This step is thought to remove accumulated
debris, to help maintain working length, and to translate into greater”
265. Color of #6 k file?
A. White
B. Pink
C. Purple
D. Green

266. Access cavity of three canals premolar?

58
A. Trapezoidal
B. Triangular
267. Ledge Bypass?
A. Using smaller files
B. Using larger files
268. Most common adverse effect of avulsed replanted too ?
A. External replacement root resorption
A. internal inflammatory resorption
269. Endo question, patient complain from his lower molar Xray show: lower molar with
crown and radiolucency in the apex and the lateral distal surface of the root. Pain in
percussion, pocket depth distally: more than 10mm, cold test: negative ?
A. subgingival calculus
B. lateral periodontal abscess
C. asymptomatic periapical periodontitis
270. most common organism found in chronic abscess?
A. Aerobic
B. Anaerobic
C. Mixed
 Acute endo ->anaerobic bacteria
 Chronic endo -> mixed bacteria
271. how to remove gutta percha from the canal ?
A. Chloroform and h file.
272. which file is used for calcified canals ?
A. H file
B. C file
C. K file
 Maillefer C-Files are made of specially treated stainless steel for amazing stiffness and
strength. The result is easier access to challenging, calcified canals.
273. How to remove pulp stone?
A. Ultrasonic tip
 Instrument —> through tip of ultrasonic scaler
 Negotiations —> c file
 Material —> EDTA
274. Post endo treatment, 8 months periapical tissue with no radiolucency?

A. Healed
B. Healing
Healed: no radiolucency
Healing: there is radiolucency but smaller than first time

59
275. Definition of endodontic flare up?
A. Acute exacerbation of periradicular pathosis after start of RCT
276. Treatment of perforation
A. Zinc oxide
B. Glass ionomer
C. Calcium hydroxide
D. Mineral trioxide aggregate
277. Access opening for upper central incisor?
A. Circular
B. Oval
C. Triangular

278. Burs for retreatment of RCT with PFM crown:


A. transition metal Carbide for metal and Diamond for porcelain

60
1.PULPAL diagnosis:
A. Normal pulp: A clinical diagnostic category in which the pulp is symptom-free and
normally responsive to pulp testing.
B. Reversible pulpitis: A clinical diagnosis based on subjective and objective findings
indicating that the inflammation should resolve and the pulp return to normal.
C. Symptomatic irreversible pulpitis: A clinical diagnosis based on subjective and
objective findings indicating that the vital inflamed pulp is incapable of healing.
Additional descriptors: lingering thermal pain, spontaneous pain, referred pain.
D. Asymptomatic irreversible pulpitis: A clinical diagnosis based on subjective and
objective findings indicating that the vital inflamed pulp is incapable of healing.
Additional descriptors: no clinical symptoms but inflammation produced by caries,
caries excavation, trauma.
E. Pulp necrosis: A clinical diagnostic category indicating death of the dental pulp. The
pulp is usually nonresponsive to pulp testing.
F. Previously treated: A clinical diagnostic category indicating that the tooth has been
endodontically treated, and the canals are obturated with various filling materials other
than intracanal medicaments.
G. Previously initiated therapy: A clinical diagnostic category indicating that the tooth
has been previously treated by partial endodontic therapy (eg, pulpotomy, pulpectomy).
2.APICAL diagnosis:
A. Normal apical tissues: Teeth with normal periradicular tissues that are not sensitive to
percussion or palpation testing. The lamina dura surrounding the root is intact, and the
periodontal ligament space is uniform.
B. Symptomatic apical periodontitis: Inflammation, usually of the apical periodontium,
producing clinical symptoms including a painful response to biting and/or percussion or
palpation. It might or might not be associated with an apical radiolucent area.
C. Asymptomatic apical periodontitis: Inflammation and destruction of apical
periodontium that is of pulpal origin, appears as an apical radiolucent area, and does not
produce clinical symptoms.
D. Acute apical abscess: An inflammatory reaction to pulpal infection and necrosis
characterized by rapid onset, spontaneous pain, tenderness of the tooth to pressure, pus
formation, and swelling of associated tissues.
E. Chronic apical abscess: An inflammatory reaction to pulpal infection and necrosis
characterized by gradual onset, little or no discomfort, and the intermittent discharge of
pus through an associated sinus tract.
F. Condensing osteitis: Diffuse radiopaque lesion representing a localized bony reaction
to a low-grade inflammatory stimulus, usually seen at apex of tooth.
3.☀KEY WORDS:
A. REVERSABLE:
 seconds (pulp return normal)

61
 not spontaneous pain
 discomfort (cold, sweet) then go away
B. SYMPTOMATIC IRRIVERSABLE PULPITIS:
 sharp pain on thermal test
 discomfort to percussion
 incapable of heat
 spontaneous pain
C. ASYMPTOMATIC IRRIVERSABLE PULPITIS:
 response normally to thermal test
 Caries / trauma.
D. SYMPTOMATIC APICAL PERIODONTITIS:
 painful (biting, percussion, palpation)
E. ASYMPTOMATIC APICAL PERIODONTITIS:
 No pain
 No clinical pic
 RL
F. ACUTE APICAL ABSCESS:
 sever spontaneous pain
 fever
 lymphadenopathy
 pus + swelling
 associated tissue
 rapid onset
G. CRONIC APICAL ABSCESS:
 little or no discomfort
 pus
 associated sinus track
 gradual onset
H. CONDENSING OSTEITIS:
 RO
 usually seen at apex of tooth
4.Acute apical abscesses no radiolucency’s? infection did not have enough time to
demineralize cortical bone, it takes 30%-50% of bone must altered to be visible.
5.Dentinal pain: caused by outflow in dentinal tubules, A-delta fibers, sharp pain, cold,
coronally, myelinated
6.Pulp pain: C fibers, dull throbbing, heat, centrally, unmyelinated
7.referred pain: preauricular pain (behind the ears) usually from man molars
8.Percussion test = positive when inflammatory process reached proprioceptive fibers of
PDL, Pulp has no proprioceptive fibers.

62
9.cold test: dichlorodifluoromethane -30, at middle third of facial surface for 5 sec , must be
dried first
10.EPT: least indicate for vista Sensory fibers, Not for Vascular, least pulp vitality test
11.EPT = measure pulp neural responses.
12.Pulse oximetry: measure oxygen concentration in blood
13.Laser Doppler flowmetry: assess blood flow
14.Least reliable in general = heat test
15.Least reliable for PFM = electrical
16.Best for PFM = cold
17.Least reliable for pedo = electrical
18.intrapulpal anesthesia: Deposit anesthetic under pressure. Resistance (back pressure) to
injection should be felt for the success
19.pulp vitality testing: pulp vascularity
20.Best determination of crown root fracture: tooth sloth
21.Needle irrigation to WL: 2mm short
22.Granuloma = asymptomatic + non vital + no response to percussion test
23.Abscess = can be symptomatic + extreme sensitive to percussion
24.Hyperalgesia = exaggerate response to painful stimuli - allodynia painful response to non-
painful stimuli
25.Full pulpectomy indication Traumatic exposures after more than 72 hours (3 days) and
carious exposure of a young tooth with a partially developed apex
26.Ellis’s classification depend on depth and layer of the injury.
a. class l: Enamel
b. class ll: Enamel + dentin
c. class lll: enamel+ dentin + pulp
d. class lV: non vital (Kills the tooth)
e. class V: luxation (move the tooth)
f. class Vl: avulsion (kicks the tooth)
27.Ellis’s classification for tooth fracture
 0 - no trauma
 1 - simple crown fracture involving little dentin
 2 - extensive crown fracture involving considerable amount of dentin
 3- fracture involving dentin and pulp
 4 - non vital tooth and no loss of crown
 5 - total loss of tooth
 6 - root fracture with or without loss of crown
 7 - displacement of tooth without crown or root fracture
 8 - IIIV fracture of crown in mass
 9- IX trauma for primary teeth
28.artificial opening: perforation

63
29.strip perforation = due to excessive coronal flaring
30.Furcal perforation = through the pulpal floor/ perforation in furcation / interradicular
31.Furcation + Strip perforation = MTA
32.perforation sealing: immediately
33.Zipping perforation: over preparation of outer wall of apical curvature of canal
34.Perforations below the crestal bone in the coronal third of the root = poorest
prognosis.
35.gouging or perforation = Failure to analyze this penetration angle carefully Avulsed
immature tooth better prognosis than closed apex = there is a greater chance of regaining
pulp vitality after replantation
36.Sings of mishap = instrument fracture, perforation, Canal transportation, ledge
37.Cervical perforation = resin ionomer, disadvantage of MTA long setting so its not
preferable for cervical
38.Perforation near cervical area has poorer prognosis
39.Lateral perforation prognosis =
 at or above height of crest of bone > favorable
 Below the crest of bone > poor
40.Storage of avulsed tooth: best media HBSS then saliva, milk, saline
41.Resorption area contains acidic PH 4.5 -5 > Calcium hydroxide neutralized the acidic
PH
42.subluxation: loosening, no displacement but increase mobility, flexible splint 1- 2 weeks
43.Extrusion: partially extruded from socket, flexible splint 2 weeks
44.lateral luxation: displacement of tooth any direction except axially, flexible splint 6- 4
weeks, metallic sound
45.intrusion: apical displacement open apex > allow to re-erupt, closed apex > reposition
and, RCT less than 3mm observation, more than 7mm surgically reposition splint 4 -8
weeks
46.Avulsion: flexible splint 2 weeks
47.Alveolar fracture: 4 weeks splint
48.storage media = HBSS > milk > saline > saliva > water
49.External resorption: damage to cementoblastic
50.external resorption ttt: RCT + calcium hydroxide
51.internal resorption: damage to odontoblastic
52.internal resorption ttt: RCT
53.Apexogenesis: IPC, DPC, Cvek (partial pulpotomy), pulpotomy of immature permanent
tooth
54.apexification: pulpectomy of immature percent tooth
55.coronal(cervical) fracture: poor prognosis, management: extract coronal segment > ortho
reposition if root is long > restoration
56.midfoot fracture: coronal necrosis and apical vital, RCT for coronal segments only

64
57.best media in open apex immerse before reimplanting: Doxycycline, minocycline for
5min
58.VRF: CBCT for diagnosis
59.poorest avulsed tooth: closed apex >24hr
60.Sequel of avulsed tooth = Undesirable periodontal ligament reactions cause> replacement
resorption (ankylosis) or inflammatory resorption of the root
61.Etiology of internal resorption = loss of predentin
62.Trephination = in absence of welling, for drainage abscess to relive pain
63.incision = soft tissue
64.trephination = opening hard tissue
65.orthograde = coronal direction canal restoration
66.retrograde = apical direction canal restoration
67.Danger zone = distal wall of mesial root (less dentine)
68.Most accurate radiograph for endo (tooth size): paralleling technique
69.Retreatment endo: initially > Drills (Gates-Glidden or Pezo) , Solvents > chloroform
(most effective)
70.Best tech for sterilization of GP: 5.25% sodium hypochlorite for 1 min
71.Balanced forced instrumentation: balancing the cutting effect quarter turn clockwise
with slight apical pressure, then half to three-quarter turn counterclockwise
72.Recommended tech for pulpectomy of max primary incisors: facial approach
73.Predominant pathogent in primary endo: gram - obligate anaerobic (Bacteroides),
Failed endo ttt: E faecalis > Facultative bacteria
74.Ttt of orofacial infection of endo origin: penicillin’s drug class > Amoxil, Augmentin
75.If do not respond to penicillins alone? = clindamycin, metronidazole (flagyl) adds to
Amoxil = effective against orofacial infection of endo origin
76.Antibiotics decrease the effect of birth control pills
77.Gow-Gates block = indicated when unsuccessful of IANB becz of infection
78.Pulp Necrosis: (concussion least, intrusion most)
79.calcific metamorphosis (canal obliteration): extensive amounts of reparative dentine
(intrusion) causes>yellow-orange color
80.crack tooth: usually extent mesiodistally
81.tissue will not regenerate dentin
82.Sinus tract: 25#, 30# GP
83.minimum to leave GP: 4-5 mm for post prep
84.biologic consideration: appropriate angle of root-end resection is perpendicular to the
long axis of the tooth
85.Diffuse swelling: Antibiotics and analgesics should be prescribed, and the patient should
be monitored closely
86.irrigation for open apex: 0.5- 1.5% NaOCl or diluted NaOCl
87.best effective irrigation for killing E, faecalis: MTAD
88.least effective for E. faecalis: CHX
65
89.pt. came late at the end of day with sever lingering pain, management: with
irreversible pulpitis > pulp extirpation (pulpectomy) + NSAIDs
90.% Of accessory canals in apical third: 74%
91.Differentiate between pulp and periodontal abscess: Vitality test
92.metallic sound: lateral luxation + intrusion
93.Latex allergy which types of obturation material or eugenol allergy: Resilon (resin-
based filling)
94.NaOCl accident: severe pain, edema of neighboring soft tissues, edema over the injured
half of face and upper lip, profuse bleeding from root canal, profuse interstitial bleeding
with hemorrhage of skin and mucosa (ecchymosis) - management: control pain with LA
and analgesics, cold compresses after 1day warm compresses.
95.Access opening shape: maxillary premolars (oval)
96.Only tooth show 8 Endo configuration: max 2nd premolar
97.ledge: an artificial irregularity created on canal wall that impedes the placement of an
instrument to the apex
98.Thermomechanical compaction: McSpadden Compactor
99.thermoplastic injection technique: Obtura Il (internal resorption)
100.thermoplasticized technique: open apex (external resorption)
101.warm vertical compaction + continuous wave compaction technique: System B
102.Carrier-Based Gutta-Percha: thermafil + pro taper + SuccessFil
103.fibers not found in 6-12 years: apical fibers
104.Tug back: resistance of cone GP in apex
105.Endo ice temperature: -26.2°C
106.Remove silver points: Stieglitz pliers and also used to remove broken instrument
107.Factors affecting EPT test:
 false +: partial necrosis - anxiety - improper isolation - contact with metal
 False -: obliteration - recently traumatize - immature apex - increase threshold
108.cells migrate to periodontal pocket: oral epithelium + gingival connective tissue +
alveolar bone + PDL
109.Revascularization:
A. medicated with triantibiotic paste
B. over instrumentation
C. The subsequent blood clot is overlaid with MTA, forming a scaffold for invasion by
stem cells from the apical papilla (SCAP cells).
 canal disinfected without mechanical instrumentation but with copious irrigation
with 5.25% NaOCl,
 followed by triantibiotic (equal amounts) of ciprofloxacin, metronidazole, and
minocycline for 3 weeks
 antibiotic was rinsed out after 3 weeks, and a blood clot produced to the level of the
CEJ followed by MTA

66
110.Necrotic pulp contains: PMN leukocytes + histiocytes
111.What to after ledge formed: bypass precurve file
112.Canal transportation = removal of canal wall structure on the outside curve in the apical
half of the canal
113.ledge = dentinal shelf that is created by shaping instruments that straighten and dig into
the convex side of the canal wall
114.Gp sterilization with sodium hypo = 1 mins
115.Calcified canals = First CBCT then sharp explorer, color change in dentine, use
ultrasonic tips, statin with 1% methylene blue dye, sodium hypochlorite (champagne
bubble test) >leave NaOCL on chamber then bubbles will appears indicate position of
orifices
116.Overfilled GP = better to remove GP with heat plunger before sealer sets otherwise > No
treatment (follow up unless there is sign and symptoms)
117.Hero file = similar to H file in cross section without radial lines, non-cutting passive tip
Voxel sizes (in CBCT) = endo imaging require high spatial resolution > smallest voxel size
= the higher resolution - The absolute maximum voxel size for endodontic imaging should
be 0.2 mm
118.Flare up = acute exacerbation of periradicular pathosis - reasons: 1- over instrumentation
2- chemical irritants (CaOH, sealers , obturation) - management: prophylactic antibiotic
119.Crack + fracture = block the light transmission
120.Fungi found in persistent endodontic infections = Candida albicans
121.Intraoral periapical radiograph = 2mm of bone below root should be visible\
122.Reason for endodontically treated posterior teeth fracture = loss of coronal structure
123.“Resistance to fracture reduced in MOD cavities”
124.Location of second canal in mandibular canine = Lingual
125.root canal preparation should stop at = minor diameter
126.Obturation should be 1-2 mm short of the radiographic apex
127.Pulp stone removed by = ultrasonic scaler
128.action of inflammation which causes pulp necrosis = vascular dilation
129.Tug back of gutta percha = to prevent excessive obturation materials into PDL
130.Root canal treated tooth and exposed to oral environment When you should do re-
endo = after 2 - 3 months
131.shape of access cavity for the mandibular first molar= rhomboid or trapezoid
132.Gingival diseases of specific bacterial origin =
 Neisseria gonorrhoeae
 Treponema pallidum
 Streptococcus species
133.Which disease accumulated glycation end-products (AGEs) in gingiva takes place? =
diabetes … the formation of AGEs occurs at normal glucose levels, but in hyperglycemic
AGE formation is excessive

67
134.Dental/pulp trauma = is intrinsic stains
135.positive-pressure irrigation methods = cause risk of expressing debris or solution out of
the apex (apical extrusion of irrigation i.e sodium hypochlorite accident)
136.Location of MB2 = mesial and palatal to MB1
137.Yellow crown discoloration after Endo = pulp obliteration (calcification)
138.K file: square shape st steel
139.Reamer: triangle shape st steel
140.K-flex: rhomboid or diamond-shape st steel
141.flex R: triangle
142.H file: round
143.K files: reaming motion (clockwise rotation)
144.H files: filing motion (push and pull motion)
145.more flexible file: K-reamer
146.RC-prep: canal lubricant, EDTA + urea peroxide + glycol-based Glyde (wax-based)
147.NaOCl: remove organic, proteolytic irrigation (breaking down proteins)
148.EDTA: remove inorganic, chelating agents
149.CHX: effective against E.faecalus organisms ass with failed endo ttt
150.All sealer cements: highly toxic when freshly mixed > reduced on setting
151.Profile rotary: rounded tip + U shaped flute Cross section
152.Normal tissue pH (7.4), in presence of inflammation decreases to 4.5 to 5.5
153.Phentolamine (oraVersa, Septodent) =local injection to reverse effective of local
anesthesia
154.11 or 12 = best scalpel for drainage of endo abscess
155.ss hand files = .02 taper
156.Ni-Ti rotary files = .04 or .06 taper
157.NaOCl = dissolves organic material
158.EDTA = dissolves inorganic materials
159.Chloroform = dissolves GP in retreatment
160.GP and sealer based = ZOE
161.Calcium hydroxide pH: 12.5
162.lentulo spiral: sealer in endo
163.NaOCl mixed with CHX: cause orange-brown occludes the dentinal tubules known as
parachloroaniline (PCA)
164.CHX mixed with EDTA: white precipitate formation
165.EDTA: Ethylenediamine Tetraacetic acid 17%
166.silver points contact tissue fluids or saliva: they corrode
167.resorbable sealer: ZOE
168.Calcium hydroxide PH: 11
169.AH plus: not released Formaldehyde., AH-26: releases Formaldehyde
170.AH-26, AH plus: epoxy resin sealers
171.Formaldehyde: called formocresol + formalin, bactericidal
68
172.MTA: for its property to reduce external root resorption
173.component of ledermix which use in endo to reduce pain: Triamcinolone acetonide
(steroid) and also ledermix may slow down resorptive processes after severe traumatic
injuries to the dentition
174.Gates Glidden sizes:
 #1 = 0.5 mm, 50
 #2 = 0.7 mm, 70
 #3 = 0.9 mm, 90
 #4 = 1.10 mm, 110
 #5 = 1.3 mm, 130
 #6 = 1.5 mm, 150
175.Tapper of K file: 0.02mm
176.Solvent for GP: chloroform
177.AH plus: working time 4 hrs., sitting time 8 hrs.
178.NaOCl: PH >11
179.Bismuth Oxide: Radioopacifier in MTA
180.Gutta percha:
 advantages: plasticity, ease of manipulation, minimal toxicity, radiopacity, and ease
of removal with heat or solvents.
 Disadvantages: lack of adhesion to dentin and, when heated, shrinkage upon cooling
181.Irrigation causes protein coagulation: eugenol - formocresol
182.Cavit: not used with Vital tooth, used as temporary materials in RCT
183.Most effective solvent and fastest and low risk: chloroform
184.Ledermix (corticosteroid antibiotic paste): contains triamcinolone acetonide 1% as an
anti-inflammatory agent - intracranial medicaments - pH 8.13
185.Formocresol: pH 12.45
186.Sodium hypochlorite: pH 11 to 12
187.Calcium hydroxide (Calyxl): pH 10 to 14
188.AH plus: pH 7.81 to 7.17 at 3 hours to 240 hours
189.Antibiotic-corticosteroid paste (Ledermix): pH 8.13
190.Eugenol: pH 4.34
191.CHX is a strongly basic: pH 5.5
192.triantibiotic paste: (metronidazole, minocycline, and ciprofloxacin)
193.A patency file = small K-file #10 or #15 passively extended slightly beyond the apical
foramen. Use for most rotary to remove accumulated debris
194.NiTi file less incident of ledge: non cutting tip
195.Peso drills = coronal flaring + post preparation.
196.MTA = repair the perforation even the cavity is contaminated with blood, its
biocompatible
197.DG-16 explorer = locate canal orifice and determine canal angulation

69
198.Explorer = detect root smoothing
199.most sealer has problem in long term stability = Calcium hydroxide based
200.GP = 20% gutta percha - 75% ZOE - radio pacifiers – plasticizers
201.Length of D16 = 0.32
202.Taper at D16= 0.32 + (file number) = Answer
203.MTA =
 Advantages: good seal + antimicrobial + set in presence of moist and blood
 Disadvantage: cause pulp obliteration +long setting time
 Main component: tricalcium silicate + tricalcium aluminate
204.Gates-Glidden drill = limited to straight canals
205.diameter of Transmetal Carbide bur = 1.6mm
206.IKI (Iodine potassium iodide) = is root canal medicaments (disinfectant) - able to kill
CaOH resistant bacteria - do not use it with pt. has iodine allergy
207.Ledermix = inhibition of the ribosomal protein synthesis / inhibition of resorption
208.D5=0.20 + (0.1)
Important files : https://drive.google.com/drive/folders/1lUQu9VDync9f-
fN5cDdtJOs2XnvjgkY9?usp=sharing

70

You might also like